You are on page 1of 48

PROFESSIONAL RESPONSIBILITY OUTLINE

SPRING 2014

REGULATING COMPETENCE, CHARACTER, & FITNESS

Who regulates?
 The court has the inherent and exclusive authority to regulate practice of law. The Court
regulates:
o Admission to the bar
o Licenses
o Adopts the professional rules of conduct
o Legislation can regulate, but the Judge may void it
 Self-Regulate (see Rule 8.3)

What is a professional?
 Educated individuals devoted to public service in an autonomous, self-regulated group
 View professionals by their qualifications, status, and kinds of work
 Your reputation is your most important professional asset b/c get clients mainly from
referrals

4 Basic Duties of Any Attorney


(1) Competence
(2) Communication (this is the one that gets lawyers in the most trouble)
(3) Confidentiality
(4) Conflict-free representation

What is the practice of law?


 Appearing in Court
 Giving Legal advice (note: giving legal advice vs. giving legal info is a fine line; giving
someone a referral is NOT legal advice).
o In Re CreasyStands for proposition that Courts have the authority to regulate
and define the practice of law maybe even over legislative actions. Here Creasy
had been disbarred already and was found to be practicing law even though it
appeared legislature permitted what he was doing. Did not matter and court still
found him in contempt. Court found that practice did not have to be in front of a
court or a tribunal and even going before private arbitration proceeding counted.
Further what good was previous disbarment order if Creasy could still practice
law and Disciplinary council had no authority over him?

What sources of law regulate attorneys?


 Rules of Professional Conduct adopted by highest court in state
o Consequence to Atty: state atty disciplinary agency imposes discipline on atty for
incompetence
 Court rules of civil or criminal procedure
o Consequence to Atty: atty seeks relief from error or is subject to sanction

1
 Inherent power of the court
o Consequence to Atty: Trial level ct holds atty in contempt or requires atty forfeit
fees
 General Common Law
o Consequence to Atty: Client sues for damages (under tort of K suit for
malpractice or suit for breach of fiduciary duty)
 Statutes
o Consequence to Atty: clients may sue for damages (under codified CL standards
or under additional legislative bases, such as consumer protection laws)
 Malpractice Insurance Contract
o Consequence to Atty: insurer increases malpractice rates or declines coverage
 Reputational Regulation
o Consequence to Atty: client fires atty and doesn’t pay for the representation.
Client, other atty’s, and court will not refer other clients

**Note: Competency is regulated very heavily in the rules of professional responsibility

**Remember: Nothing AUTOMATICALLY = Disbarment

ADMISSION V. DISCIPLINE

Admission
 Burden of Proof: burden is on the applicant
 Who admits attorneys?  The court admits attorneys to the bar
 Conduct need not be something that would violate a rule of professional conduct in order
to provide a basis for determining that an applicant lacks good moral character and fitness
to practice law.
o Lane v. Bar Commission of NE State Bar Association Questionable
violations of the rules of professional conduct may be looked at more severely in
the application stage. Here guy was coarse, rude, sexist, hostile and maybe
threatening on more than one occasion. On one occasion, when Lane was helping
out w/ clinic, Lane threatened a professor of the clinic when she wouldn’t let him
step past the bar and said that he would not work with women students. On
second occasion at a BAR-BRI review course, Lane left 10 minutes early and then
returned and demanded to know who had stolen his keys. Lane used strong and
profane language in accusing the students. In addition, his employment history
indicated that he had outbursts in the lobby while filling out an application and
that he was very rude to female employees and that he walked off one job,
allegedly telling one supervisor to have all his employees see a psychiatrist.
Court held this was grounds for a proper denial of admission to bar. Conduct
reflected poorly on the legal profession and this is not the behavior acceptable of
counsel. (also guy would not have been admitted due to his failure to adequately
answer all questions presented to him upon application (would violate lack of

2
candor to tribunal Rule 3.3 and exhibits flaws in trustworthiness, honesty and
reliability of Rule 8.4(b)&(c), which provide a separate basis for not admitting)).
o In Re Whitworth Whitworth was suspended for 2 years b/c of his addiction to
meth and alcohol. Whitworth voluntarily told the disciplinary counsel of this b/c
it was his duty to report it (originally Whitworth was being seen by the
disciplinary counsel b/c of other misconduct – the most frequent violation being
his failure to respond to the Bar’s allegations). Was then suspended for 2 yrs, but
said he could be reinstated in 2 years by demonstrating his sobriety and by
meeting all the requirements for Rule 11 reinstatement. Although readmission
which requires higher standard than even admittance this shows burden is still on
applicant. In readmittance must consider (1) Present moral fitness to practice; (2)
Consciousness of previous wrongs; (3) Extent of applicants rehab (drugs and
alcohol mainly); (4) Seriousness of original misconduct; (5) Conduct after
discipline; (6) Time since previous discipline; (7) Character, Maturity and
Experience of applicant; (8) Present legal competence. Applicant must show
clear and convincing evidence of fitness to practice law to be readmitted. Here,
Whitworth was able to be reinstated b/c he candid about his addictions, was extra
involved in AA, and passed all random drugs tests.
o In Re Heart: applicant who sent email to a member of the board of bar examiners
at 3am about asking for special favors was improper and she was denied the
opportunity to sit for the bar at that time. Even though it may be innocent conduct
it was still unacceptable and improper.
 Why do we regulate admission to the practice of law?
o To require minimum levels of competence
o Create affirmative standards of conduct or reiterate requirements of general law
o Protect clients who do not know the law
o Protect clients from fraud, mishandling of funds, betrayal of confidences, etc.
o Protect courts and the administration of justice from subordination,
misrepresentation, bribery, etc.
o Protect image/reputation of legal profession
o Protect profession from eternal regulation
o To control supply of legal services

*Tougher standard in admittance because it is better to protect the public from admitting those
not competent or fit to practice than accidentally excluding few who are competent

Disciplinary Matters
 Burden of Proof: Burden is on the state (is not beyond a reasonable doubt though)
 Purpose of discipline is not punishment – purpose is protection of the public
 When it is a disciplinary matter, the Court Rules are the binding authority
o Note: courts promulgate rules by court orders
 4 Ways to Get to S.C. on a Disciplinary Matter:

3
1. Complainant can reject disciplinary counsel’s recommendation
2. Disciplinary counsel can reject disciplinary counsel’s recommendation
3. Respondent counsel can reject disciplinary counsel’s recommendation
4. Court can reject disciplinary counsel’s recommendation
 4 areas of practice that are at highest risk of complaint:
o Domestic
o Criminal
o Torts
o Estates
 Most common reasons for malpractice and discipline are grounded in management
(missing deadlines, having too many cases, etc.); client relationships (poor
communication); and attorney self-management (stress, substance abuse, other
addictions).
 An attorney may be disciplined for a single mistake caused by mere negligence – BUT,
as a practical matter, disciplinary actions are almost never based on a single instance of
mere negligence
 How NOT to respond to a disciplinary complaint:
o Lie
o Ignore
o Attack/contact the complainant or the process
o Quibble/rationalize/make excuses
o Try to shift responsibility
 Practical Influences on Administration of a Discipline System
 Degree of risk to the public
 Costs of investigation and prosecution
 Alternatives to discipline (e.g., lawyers assistance programs)

SANCTIONS

Sanctions
 Aggravating Circumstances for Choosing a Sanction
o Prior Discipline
o Dishonest or Selfish Motive for actions
 Mitigating C Circumstances for Choosing a Sanction:
o Cooperation
o Remorse
 Baseline Elements for Choosing a Sanction
o Duty Violated
o Lawyers mental state (level of intent), and
o The extent of injury or potential injury

NOTE: What can you do to protect yourself from a malpractice suit?


 Carry malpractice insurance
o Can get real reasonable rates through the MO Bar Plan

4
 Take notes – be organized

PREAMBLE
A Lawyer’s Responsibilities
 Lawyers perform various functions:
o Advisor: provides client w/ an informed understanding of the client’s legal rights
and obligations and explains their practical implications
o Advocate: zealously asserts the client’s position under the rules of the adversary
system
o Negotiator: seeks a result advantageous to the client but consistent w/
requirements of honest dealings w/ others
o Evaluator: examining a client’s legal affairs and reporting about them to the client
or to others
 Lawyer should be competent, prompt, & diligent
 A lawyer can be sure that preserving client confidences ordinarily serves the public
interest b/c ppl are more likely to seek legal advice, and thereby heed their legal
obligations, when they know their communications will be private
 Legal profession is largely self-governing

Scope
 Rules of Professional Conduct are rules of reason
 Violation of a rule should not itself give rise to a cause of action against a lawyer nor
should it create any presumption in such a case that a legal duty has been breached
 Violation of a rule also does not necessarily warrant any other nondisciplinary remedy,
such as disqualification of a lawyer in pending litigation

CLIENT-LAWYER RELATIONSHIP

Rule 1.0 Terminology


(a) "Belief" or "believes" denotes that the person involved actually supposed the fact in question
to be true. A person's belief may be inferred from circumstances.

(b) "Confirmed in writing," when used in reference to the informed consent of a person, denotes
informed consent that is given in writing by the person or a writing that a lawyer promptly
transmits to the person confirming an oral informed consent. See paragraph (e) for the definition
of "informed consent." If it is not feasible to obtain or transmit the writing at the time the person
gives informed consent, then the lawyer must obtain or transmit it within a reasonable time
thereafter.

(c) "Firm" or "law firm" denotes a lawyer or lawyers in a law partnership, professional


corporation, sole proprietorship or other association authorized to practice law; or lawyers
employed in a legal services organization or the legal department of a corporation or other
organization.

(d)  "Fraud" or "fraudulent" denotes conduct that is fraudulent under the substantive or
procedural law of the applicable jurisdiction and has a purpose to deceive.

5
NOTE: rules defer to substantive law to define fraud; but to constitute fraud under the
rules conduct must have a purpose to deceive! A negligent misrepresentation or negligent
failure to apprise another of relevant information CANNOT constitute as fraud under the
rules

(e) "Informed consent" denotes the agreement by a person to a proposed course of conduct after
the lawyer has communicated adequate information and explanation about the material risks of
and reasonably available alternatives to the proposed course of conduct.

(f)   "Knowingly," "known," or "knows" denotes actual knowledge of the fact in question. A
person's knowledge may be inferred from circumstances.

(g) "Partner" denotes a member of a partnership, a shareholder in a law firm organized as a


professional corporation, or a member of an association authorized to practice law.

(h) "Reasonable" or "reasonably" when used in relation to conduct by a lawyer denotes the


conduct of a reasonably prudent and competent lawyer.

(i)   "Reasonable belief" or "reasonably believes" when used in reference to a lawyer denotes
that the lawyer believes the matter in question and that the circumstances are such that the
belief is reasonable.

(j)  "Reasonably should know" when used in reference to a lawyer denotes that a lawyer of
reasonable prudence and competence would ascertain the matter in question.

(k) "Screened" denotes the isolation of a lawyer from any participation in a matter through the
timely imposition of procedures within a firm that are reasonably adequate under the
circumstances to protect information that the isolated lawyer is obligated to protect under these
Rules or other law.

(l)  "Substantial" when used in reference to degree or extent denotes a material matter of clear
and weighty importance.

(m) "Tribunal" denotes a court, an arbitrator in a binding arbitration proceeding or a legislative


body, administrative agency or other body acting in an adjudicative capacity. A legislative body,
administrative agency or other body acts in an adjudicative capacity when a neutral official, after
the presentation of evidence or legal argument by a party or parties, will render a binding legal
judgment directly affecting a party's interests in a particular matter.

(n) "Writing" or "written" denotes a tangible or electronic record of a communication or


representation, including handwriting, typewriting, printing, photostating, photography, audio or
videorecording, and electronic communications. A "signed" writing includes an electronic sound,
symbol or process attached to or logically associated with a writing and executed or adopted by a
person with the intent to sign the writing.

ETHICAL DUTIES IN THE INITIAL INTERVIEW (see full rules below):

6
 1.1 – are you competent to represent this client?
 1.2 – what does your client expect?
 1.3 – do you have time?
 1.4 – can you communicate with this client effectively?
 1.5 – can the client afford your services?
 1.18 – you have a duty of confidentiality and loyalty even to a prospective client
 2.1 – clients are owed your independent professional judgment (even if they don’t like it)

WAYS SOMEONE BECOMES YOUR CLIENT:


 Your appointed
o Moment that the court signs the order, you are appointed
 Moment that you agree to represent client
 Reasonable assumption
o Ex: might give them advice and client might reasonably rely on it

Rule 1.1 Competence


A lawyer shall provide competent representation to a client. Competent representation requires
the legal knowledge, skill, thoroughness and preparation reasonably necessary for the
representation.

Rule 1.2 Scope Of Representation And Allocation Of Authority Between Client And
Lawyer
(a) Subject to paragraphs (c) and (d), a lawyer shall abide by a client's decisions concerning the
objectives of representation and, as required by Rule 1.4, shall consult with the client as to the
means by which they are to be pursued. A lawyer may take such action on behalf of the client as
is impliedly authorized to carry out the representation. A lawyer shall abide by a client's decision
whether to settle a matter. In a criminal case, the lawyer shall abide by the client's decision, after
consultation with the lawyer, as to a plea to be entered, whether to waive jury trial and whether
the client will testify.
(b) A lawyer's representation of a client, including representation by appointment, does not
constitute an endorsement of the client's political, economic, social or moral views or activities.
(c) A lawyer may limit the scope of the representation if the limitation is reasonable under the
circumstances and the client gives informed consent.
(d) A lawyer shall not counsel a client to engage, or assist a client, in conduct that the lawyer
knows is criminal or fraudulent, but a lawyer may discuss the legal consequences of any
proposed course of conduct with a client and may counsel or assist a client to make a good faith
effort to determine the validity, scope, meaning or application of the law.

Authority
• Very few decisions that clearly are the sole authority of either the client or the attorney.
• The client has the decisions about firing the attorney, how much money to spend,
terminating litigation (through settlement or dismissal), and testifying (in a criminal
case).
• The attorney has almost no clear authority to act except the negative authority to refrain
from violating the law or assisting in violations.

7
• Almost any other example of an attorney’s authority could be subject to constraints by
the client. Unclear how much authority an attorney may exercise without first consulting
a client.
• The best resolution of the lack of clarity is discussing the allocation of authority with
your client from the beginning and thoroughly understanding their objectives and
interests.
Engagement Agreement Clause (Coleman case)
 An example of what is NOT allowed in an engagement agreement clause: "To assure that
Plaintiff does not settle for too low an amount (thereby depriving Attorney of a
reasonable contingent fee), no settlement shall be accepted unless agreed to in writing by
both Attorney and Plaintiff."
 this type of clause is not allowed b/c in 1.2 atty’s are specifically instructed to
abide by the client’s decision to settle so under 8.4 this would be misconduct

Rule 1.3 Diligence


A lawyer shall act with reasonable diligence and promptness in representing a client.

When may you ignore a client’s directives?


 Rule 1.3 Comment 1: “A lawyer is not bound, however, to press for every advantage that
might be realized for a client. For example, a lawyer may have authority to exercise
professional discretion in determining the means by which a matter should be pursued.
See Rule 1.2. The lawyer's duty to act with reasonable diligence does not require the use
of offensive tactics or preclude the treating of all persons involved in the legal process
with courtesy and respect.”
Rule 1.4 Communication
(a) A lawyer shall:
(1) promptly inform the client of any decision or circumstance with respect to which the
client's informed consent, as defined in Rule 1.0(e), is required by these Rules;
(2) reasonably consult with the client about the means by which the client's objectives are
to be accomplished;
(3) keep the client reasonably informed about the status of the matter;
(4) promptly comply with reasonable requests for information; and
(5) consult with the client about any relevant limitation on the lawyer's conduct when the
lawyer knows that the client expects assistance not permitted by the Rules of Professional
Conduct or other law.
(b) A lawyer shall explain a matter to the extent reasonably necessary to permit the client to
make informed decisions regarding the representation.

Three of the ways that Rule 1.4 requires attorneys to communicate with clients are:
(1) promptly inform the client of any decision or circumstance with respect to which
the client's informed consent, as defined in Rule 1.0(e), is required by these Rules
(2) reasonably consult with the client about the means by which the client's objectives
are to be accomplished; and
(3) A lawyer shall explain a matter to the extent reasonably necessary to permit the
client to make informed decisions regarding the representation.

8
Models of Communication and Authority
 Directive (Parent, Expert)
 Collaborative (Counselor, Friend)
 Instrumental (Hired Gun, Employee)

Rule 1.5 Fees


(a) A lawyer shall not make an agreement for, charge, or collect an unreasonable fee or an
unreasonable amount for expenses. The factors (not exclusive list) to be considered in
determining the reasonableness of a fee include the following:
(1) the time and labor required, the novelty and difficulty of the questions involved, and
the skill requisite to perform the legal service properly;
(2) the likelihood, if apparent to the client, that the acceptance of the particular
employment will preclude other employment by the lawyer;
(3) the fee customarily charged in the locality for similar legal services;
(4) the amount involved and the results obtained;
(5) the time limitations imposed by the client or by the circumstances;
(6) the nature and length of the professional relationship with the client;
(7) the experience, reputation, and ability of the lawyer or lawyers performing the
services; and
(8) whether the fee is fixed or contingent.
(b) The scope of the representation and the basis or rate of the fee and expenses for which the
client will be responsible shall be communicated to the client, preferably in writing, before or
within a reasonable time after commencing the representation, except when the lawyer will
charge a regularly represented client on the same basis or rate. Any changes in the basis or rate
of the fee or expenses shall also be communicated to the client.
(c) A fee may be contingent on the outcome of the matter for which the service is rendered,
except in a matter in which a contingent fee is prohibited by paragraph (d) or other law. A
contingent fee agreement shall be in a writing signed by the client and shall state the method by
which the fee is to be determined, including the percentage or percentages that shall accrue to the
lawyer in the event of settlement, trial or appeal; litigation and other expenses to be deducted
from the recovery; and whether such expenses are to be deducted before or after the contingent
fee is calculated. The agreement must clearly notify the client of any expenses for which the
client will be liable whether or not the client is the prevailing party. Upon conclusion of a
contingent fee matter, the lawyer shall provide the client with a written statement stating the
outcome of the matter and, if there is a recovery, showing the remittance to the client and the
method of its determination.
(d) A lawyer shall not enter into an arrangement for, charge, or collect:
(1) any fee in a domestic relations matter, the payment or amount of which is contingent
upon the securing of a divorce or upon the amount of alimony or support, or property settlement
in lieu thereof; or
(2) a contingent fee for representing a defendant in a criminal case.
(e) A division of a fee between lawyers who are not in the same firm may be made only if:
(1) the division is in proportion to the services performed by each lawyer or each lawyer
assumes joint responsibility for the representation;

9
(2) the client agrees to the arrangement, including the share each lawyer will receive, and
the agreement is confirmed in writing; and
(3) the total fee is reasonable.

Fees Raise Multiple Ethical Issues:


 Competence: Charging Enough
 What is a fee?
 How can you be paid?
How do you determine what is enough?
 Communication
 Confidentiality
 Conflicts of Interest

Regulation of Fee Amounts


 Discipline
 Litigation to enforce fees
 Statutory fee-shifting statutes
 Fee schedules
 Client fee auditors
 Criminal and civil liability

What is a “reasonable” fee


 The test employed by the ABA codes to measure reasonableness is so vague that it is
virtually useless. The test invokes no fewer than eight multi-part factors—and the list is
expressly non-exclusive. Accordingly, on almost any given set of facts, application of the
factors to a fee could sustain a reasonable argument that the fee is justified and a
plausible argument could also be made that the fee should have been different.

Practical Guidelines for Setting Fees


 Negotiated
 Legal
 Proportionate to work, value, and market

TRUE OR FALSE: You get what you pay for. Standards of competence are lower for pro
bono representation
 FALSE – The minimum standards of competence do not vary according to the level of
payment.
 BUT – Standards for ineffective assistance of counsel require proof of “prejudice” and
have a strong presumption of effectiveness
 BUT – Some statutes insulate attorneys from malpractice actions (e.g. some government
and appointed attorneys)

TRUE OR FALSE? An attorney may not be disciplined for a single mistake caused by mere
negligence.
 FALSE - That’s not how the rule reads

10
 BUT – as a practical matter, disciplinary actions are almost never based on a single
instance of mere negligence
 Missouri’s Rule 8.3’s requirement of reporting is not triggered by a single act of simple
negligence

TRUE OR FALSE? The more you are an expert in an area of law, the lower your malpractice
risk.
 FALSE – the most common reasons for malpractice and discipline are grounded in
management (missing deadlines, having too many cases, etc.), client relationships (poor
communication), and attorney self-management (stress, substance abuse, other
addictions)

Rule 1.6 Confidentiality Of Information


(a) A lawyer shall not reveal information relating to the representation of a client unless the
client gives informed consent, the disclosure is impliedly authorized in order to carry out the
representation or the disclosure is permitted by paragraph (b).
(b) A lawyer may reveal information relating to the representation of a client to the extent the
lawyer reasonably believes necessary:
(1) to prevent reasonably certain death or substantial bodily harm;
(2) to prevent the client from committing a crime or fraud that is reasonably certain to
result in substantial injury to the financial interests or property of another and in furtherance of
which the client has used or is using the lawyer's services;
(3) to prevent, mitigate or rectify substantial injury to the financial interests or property of
another that is reasonably certain to result or has resulted from the client's commission of a crime
or fraud in furtherance of which the client has used the lawyer's services;
(4) to secure legal advice about the lawyer's compliance with these Rules;
(5) to establish a claim or defense on behalf of the lawyer in a controversy between the
lawyer and the client, to establish a defense to a criminal charge or civil claim against the lawyer
based upon conduct in which the client was involved, or to respond to allegations in any
proceeding concerning the lawyer's representation of the client;
(6) to comply with other law or a court order; or
(7) to detect and resolve conflicts of interest arising from the lawyer’s change of
employment or from changes in the composition or ownership of a firm, but only if the revealed
information would not compromise the attorney-client privilege or otherwise prejudice the client.
(c) A lawyer shall make reasonable efforts to prevent the inadvertent or unauthorized disclosure
of, or unauthorized access to, information relating to the representation of a client.

Who does Rule 1.6 protect? (primarily protects the attorney!)


 Third persons who could be harmed
 The client - gives the attorney some discretion to be able to dissuade the client
 The attorney – protects from being in the dilemma of choosing between liability for
nondisclosure and sanction for disclosure

Exceptions to Confidentiality: Waivers and Consent (express & implied)

11
Duty of Confidentiality
 Is very broad!
 Confidentiality applies to ALL information relating to representation!!
 Confidentiality Exceptions:
o Client’s “informed consent”
o Implied authorization
o A disclosure does not waive that duty
o Crime-Fraud Exception:
 Exception to the ethical duty permits discretionary disclosure regarding
future or ongoing crimes/fraud
 Requires that information be gained as part of the representation; ethical duty
requires that the attorneys’ services have or will be used

Attorney-Client Privilege
 Is very narrow!
 This privilege is NOT meant to protect information
 The communication b/w the attorney and the client is what is protected; however, not all
communications b/w a client and lawyer are privileged.
 The privilege only insulates communications that assist the atty to formulate and render
legal advice – it applies only to disclosures necessary to obtain informed legal advice
which might not have been made absent the privilege.
 Privilege belongs to the client
 Client may agree to waive the privilege
 Attorney has the power (even if he lacks the authority) to waive the client’s privilege
 Disclosure outside the privileged relationship waives the privilege as to that
communication
 Crime-Fraud Exception (an exception to the privilege for communications about client
wrongdoing)
o Applies only if communications are privileged
o Requires a completed crime (or fraud)
o Compels the testimony if the client used or sought to use the attorney’s services or
advice to further that crime
o **Difficult proof requirements
o Basically: exception to privilege permits compelled disclosure regarding past
crimes/frauds.
o Requires that information be gained as part of the representation; ethical duty
requires that the attorneys’ services have or will be used
o NOTE: Jurisdictions vary in their interpretations of this exception! Do you
research!
McClure v. Thompson
 Atty Mecca appointed to represent McClure, who had been arrested for murder of mom
and disappearance of her 2 kids. Mecca met w/ McClure several times at jail and each
time McClure would tell Mecca little pieces of info regarding the children. At one point
McClure even drew a map of where the children would be and gave it to Mecca

12
 Mecca never knew if children were alive or dead – he suspected they were dead but
wasn’t sure. At one point McClure even said that “Satan killed Carol” but “Jesus saved
the kids.”
 Mecca then turned over the map to the Sheriff’s dept. McClure contended that this was
ineffective assistance and that Mecca should not have handed over the map b/c it was
confidential information. Mecca claims that it was reasonable for him to reveal the info
on order to prevent imminent death or substantial bodily harm – this exception requires
that an atty reveal confidences only to the extent that he reasonably believes necessary to
prevent those criminal acts and imminent harms.
 Ct. holds the guiding rule for purposes of the exception for preventing criminal
acts is objective reasonableness in light of the surrounding circumstances
 Is a close case but Mecca made the disclosure reasonably believing it was necessary to
prevent the client from committing a criminal act that Mecca believed was lkely to result
in imminent death or substantial bodily harm. 1.6(b)(1).

REMEMBER: Disclosures waive privilege if disclose COMMUNICATIONS; but the opposite


is not true  waiver of privilege does NOT destroy confidentiality

 Can inadvertently waive both confidentiality and Privilege by disclosing documents to


others however this is not automatic
 If inadvertent disclosure takes place Fed.R.Evid. 502 applies (502 is Attorney-Client
privilege and work product; limitations on waiver)
o Peterson v. BernardiCounsel accidentally disclosed privileged and work
product material to opposing counsel. In order to not have waived privilege the
one who disclosed privilege must prove:
1. Info or document was privileged or protected
2. Discloser must have taken reasonable steps to prevent disclosure
3. Discloser must have promptly taken reasonable steps to rectify disclosures
This will be a fact specific inquiry. Here attorney did not present evidence of the
exact steps he took to prevent disclosure since he only mentioned one step he took
and that was that he conducted a “privilege review.” However, atty did not state
when his review occurred, how much time he took to review the documents, what
documents were reviewed, and other basic details of the review process.
Furthermore, his excuse that he was time crunched did not fly since he had
months to prepare discovery. He also waited awhile after disclosing to ask for
documents back. However, court did allow for a few of the documents not to be
waived as they were clearly work product and privileged (shows even if factors
not met court has discretion to still protect some documents)

Client Property & Financial Crimes and Frauds

What if your services have been used for fraud?


 In most jurisdictions today you can disclose in order to prevent substantial financial or
property harm IF YOUR SERVICES HAVE BEEN USED TO FURTHER THAT
CRIME OR FRAUD

13
 Cf. Rule 1.16(b) permission to withdraw
 May “raise the red flag” (i.e., disclaim any documents)

Rule 1.6(b)(2)&(3) -- What is FRAUD?


 Blacks Law Dictionary: “All multifarious means which human ingenuity can devise, and
which are resorted to by one individual to get an advantage over another by false
suggestions or suppression of the truth. It includes all surprises, tricks, cunning or
dissembling (disguising, concealing), and any unfair way which another is cheated."
 Fraud is a doctrine that can be found in the law of: Torts (misrepresentation, deceit);
Contracts (fraud/misrepresentation as a defense to contract formation or enforcement);
Criminal law statutes (mail fraud, theft by deceit); and Restitution (constructive trusts)

State v. Gonzales
 Atty permitted to withdraw under 1.16 because she had knowledge that former client was
going to commit perjury Ms. Gonzales’s case and this would materially limit her ability
to cross-examine the witness (1.7(a)(2) would create a conflict). State then subpoenaed
atty to testify about the former client’s expressed intent to commit perjury. Atty filed a
motion to quash the subpoena arguing that answering the prosecutor’s questions would
violate her duty of client confidentiality under 1.6 and 1.9, exposing her to disciplinary
action.
 Prosecutor tried to use Rule 3.8 to compel atty to testify. 3.8 prohibits a prosecutor from
subpoenaing a defense lawyer in a “criminal proceeding to present evidence about a past
or present client UNLESS the prosecutor reasonably believes
o (1) the info sought is not protected from disclosure by any applicable privilege;
o (2) evidence sought is essential to the successful completion of an ongoing
investigation or prosecution; and
o (3) there is no other feasible alternative to obtain the info.”
 Atty challenged first factor – that the a/c privilege did not apply b/c of the crime-fraud
exception. Atty said there was nothing other than the former client’s communication
itself to demonstrate the client’s intention to obtain legal advice to enable or assist in the
planned perjury, which is inadequate to qualify under KSA 60-426(b)(1), which requires
“sufficient evidence, aside from the communication…to warrant a finding that the legal
service was sought or obtained in order to enable or aid the commission or planning of a
crime.”
 Was no such evidence beyond the former client’s communication itself existed and did
not support an inference that the former client sough legal advice to further a crime or
planned crime. Atty NOT required to testify!

1.6 Variations

1.6 Variations Across Time:


1983 Model Code of Professional Responsibility
DR 4-101 -Preservation of Confidences and Secrets of a Client.10
…. (C) -A lawyer may reveal:
…. (3) -The intention of his client to commit a crime and the information necessary to prevent

14
the crime

1984 Model Rules of Professional Conduct


Rule 1.6(b) A lawyer may reveal such information to the extent the lawyer reasonably believes
necessary:
(1) to prevent the client from committing a criminal act that the lawyer believes is likely to result
in imminent death or substantial bodily harm; …
 
Current Model Rules of Professional Conduct
Rule 1.6(b) A lawyer may reveal information relating to the representation of a client to the
extent the lawyer reasonably believes necessary:
(1) to prevent reasonably certain death or substantial bodily harm…

1.6 Variations Across Jurisdictions:


Missouri Rules of Professional Conduct, Mo. Sup. Ct. Rule 4-1.6(b) 
(b) A lawyer may reveal information relating to the representation of a client to the extent the
lawyer reasonably believes necessary:  
(1) to prevent death or substantial bodily harm that is reasonably certain to occur;
(This is the very similar to the current Model Rule 1.6(b)(1) – there is no equivalent to 1.6(b)
(2)or (3))conff

Kansas Rules of Professional Conduct, Ks. Sup. Ct. Rule 1.6(b)  


A lawyer may reveal such information to the extent the lawyer reasonably believes necessary:
(1) To prevent the client from committing a crime…
(This is the same as the 1983 ABA Code of Professional Responsibility. There are no other
exceptions to prevent harm to third persons)

Iowa Rules of Professional Conduct, Iowa Sup. Ct. Rule 32:1.6  


In addition to the language of Model Rule 1.6(b)(1)-(6), Iowa includes this provision: 
(c) A lawyer shall reveal information relating to the representation of a client to the extent the
lawyer reasonably believes necessary to prevent imminent death or substantial bodily harm

Rule 1.7 Conflict Of Interest: Current Clients


(a) Except as provided in paragraph (b), a lawyer shall not represent a client if the representation
involves a concurrent conflict of interest. A concurrent conflict of interest exists if:
(1) the representation of one client will be directly adverse to another client; or
(2) there is a significant risk that the representation of one or more clients will be
materially limited by the lawyer's responsibilities to another client, a former client or a third
person or by a personal interest of the lawyer.
(b) Notwithstanding the existence of a concurrent conflict of interest under paragraph (a), a
lawyer may represent a client if:
(1) the lawyer reasonably believes that the lawyer will be able to provide competent and
diligent representation to each affected client;
(2) the representation is not prohibited by law;
(3) the representation does not involve the assertion of a claim by one client against
another client represented by the lawyer in the same litigation or other proceeding before a

15
tribunal; and
(4) each affected client gives informed consent, confirmed in writing.

Standard: DAML (directly adverse or materially limited)


 Materially Limited
o When a conflict is possible but not actual, the mere possibility can create a
conflict because it may materially limit your representation – question is one of
degree and severity (“significant risk”)
o Any interests – whether clients, third parties, personal, professional – can
materially limit your representation.
 Directly Adverse Clients
o Suing your own client – even if only nominal, is always a conflict (loyalty) 1.7,
cmt 6
o Direct adversity in transactions – sitting across the table from your client
o Directly adverse doesn’t mean adverse in emotional or business kinds of senses
 In other words, if a client says “I can’t believe you represent this other
client – I hate that other client,” then that does NOT matter. It’s none of
your client’s business if you are representing another client if their
LEGAL interests aren’t adverse

In Re Dresser Industries, Inc.


 May a law firm sue its own client, which it concurrently represents in other unrelated
matters?
o No; and certainly not here where the motivation appears only to be the law firm’s
self-interest
 Approach to Concurrent Representation
o Dresser Court Examined the following to determine if the dual representation
would amount to impropriety:
 Appearance of impropriety in general
 NOT a standard in the Rules of Professional Responsibility 
standard is directly adverse or materially limited
 Will still see courts talking about this though
 Possibility that specific impropriety will occur
 Likelihood of public suspicion of impropriety that outweighs social
interest served by lawyer’s continued participation
o Motion to disqualify granted
 Disqualification motions are where most conflicts are played out
o Disqualification isn’t always granted when there is a
conflict b/c sometimes it is too costly and sometimes they
are being abused tactically

16
o Atty might have been able to continue his dual representation if he could have
shown some social interest to be served by his representation that would outweigh
the public perception of his impropriety.
 Rule: Cannot sue your own client, even in an unrelated matter!

Wyeth v. Abbott Laboratories


 Takes less of a categorical approach and takes a very fact based, balancing approach
 Says there is a multitude of factors to look at and says it is NOT impossible to sue your
own client and not have a conflict (although some courts have advocated a mandatory
disqualification rule, the more modern approach, when faced w/ a conflict problem, is to
carefully examine the totality of the circumstances, taking a balanced approach that
includes evaluating the impact and nature and degree of conflict).
 Ct says that moving away from a mandatory disqualification rule is not intended to
undermine 1.8, but rather, it allows for courts to fashion more equitable solutions to a
conflict problem --- court must strike a “delicate balance” b/w the competing
considerations
o On one hand, Ct must examine the potential hardships that one party will
experience if his lawyer is disqualified; but on the other hand, the ct. must weigh
the potential hardships to the adversary if counsel is permitted to proceed.
 Factors to Determine whether Disqualification is Warranted:
o Does atty have access to confidential info relevant to case at issue?
o Costs of new counsel and time to acquaint new counsel w/ facts?
o Who was responsible for creating the conflict?
o Are there two matters related in substance?
o Are both matters presently active?
o Were attorneys at issue involved in both matters?
o Are matters being handled from different offices?
o Whether the atty’s from the law firm work w/ different client representatives for
each matter?
o Relative time billed by the law firm to each matter?
 Disqualification of Howrey is not warranted here – Court is hard pressed to see how the
duty of loyalty on behalf of Howrey law firm will be compromised just b/c Howrey
represents a Wyeth entity in Europe

NOTE: In order to sue your own client, you will have to get informed consent!! (see def. of
informed consent in 1.0). And although you can represent 2 clients and have conflicts of
interest – you may have trouble.

What to include in Agreement when taking on 2 Clients that have Conflicts of Interests?
 Nature of the conflict (or your risk of conflict)
 Harms that could result from the conflict and the steps you have taken to reduce those
risks
 Reasons the client wants to waive the conflict (the benefits of the representation)
 Waiver of a/c privilege
 They can seek other representation

17
 Have them expressly waive the conflict
o Note: always want this but alone it is not enough – also need waivers of
confidentiality
 Confidentiality waivers that might be a part of this consent to conflict
 Circumstances in which you would seek to withdraw even given the client’s consent

Key Questions to Ask


 Would your representation of one client be less competent and diligent if you represent
the other client?
 Would you be likely to have confidential information from one client that could be used
to harm or advantage the other client?
 Would the courts consider your representation of one client as a breach of your loyalty to
the other client?

Pros for Concurrent Conflicts of Interest:


 Less expensive
 “Circle the wagons” consistency of theory and evidence strengthens action
 Focus on commonality rather than conflict facilitates transactions and preserves
relationships
 Client autonomy – allows clients their choice of attorney

Cons for Concurrent Conflicts of Interest:


 May be required to waive some claims or forego some theories
 Recall impact on attorney-client privilege –
 For attorneys, representation of co-clients constitutes a prime source of malpractice
claims and frequent source of discipline.
 If conflict arises later that requires withdrawal, clients, courts and attorneys are all worse
off than if the clients had begun with separate counsel or pro se representation.

MISCONCEPTIONS
 Conflicts of interest require knowledge  If I don’t know about a client in the firm
whose interest is directly adverse to my client, I don’t have a conflict of interest.
 Once I have screened a potential client for conflicts of interest and have found none, I
don’t have to worry about conflicts any more.
 If my client has signed a waiver of conflicts, I do not need to worry about disqualification
or discipline.
 If I have a conflict of interest that would prevent me from representing a particular client
in your firm, other attorneys in my firm may represent that client if I am screened from
participation in that case.

18
 If a client would be upset if they knew I was representing another client, I have a conflict
of interest that would permit that client to disqualify me from representing the other
client.
 If I represent one client, I can not represent a client that competes with that client (e.g.,
CVS and Walgreens) because they are directly adverse to one another.
 Once you advocate for a particular issue or interpretation, you may not advocate the other
side of that issue in another case.

Rule 1.8 Conflict Of Interest: Current Clients: Specific Rules


(a) A lawyer shall not enter into a business transaction with a client or knowingly acquire an
ownership, possessory, security or other pecuniary interest adverse to a client unless:
(1) the transaction and terms on which the lawyer acquires the interest are fair and
reasonable to the client and are fully disclosed and transmitted in writing in a manner that can be
reasonably understood by the client;
(2) the client is advised in writing of the desirability of seeking and is given a reasonable
opportunity to seek the advice of independent legal counsel on the transaction; and
(3) the client gives informed consent, in a writing signed by the client, to the essential
terms of the transaction and the lawyer's role in the transaction, including whether the lawyer is
representing the client in the transaction.
(b) A lawyer shall not use information relating to representation of a client to the disadvantage of
the client unless the client gives informed consent, except as permitted or required by these
Rules.
(c) A lawyer shall not solicit any substantial gift from a client, including a testamentary gift, or
prepare on behalf of a client an instrument giving the lawyer or a person related to the lawyer
any substantial gift unless the lawyer or other recipient of the gift is related to the client. For
purposes of this paragraph, related persons include a spouse, child, grandchild, parent,
grandparent or other relative or individual with whom the lawyer or the client maintains a close,
familial relationship.
(d) Prior to the conclusion of representation of a client, a lawyer shall not make or negotiate an
agreement giving the lawyer literary or media rights to a portrayal or account based in substantial
part on information relating to the representation.
(e) A lawyer shall not provide financial assistance to a client in connection with pending or
contemplated litigation, except that:
(1) a lawyer may advance court costs and expenses of litigation, the repayment of which
may be contingent on the outcome of the matter; and
(2) a lawyer representing an indigent client may pay court costs and expenses of litigation
on behalf of the client.
(f) A lawyer shall not accept compensation for representing a client from one other than the
client unless:
(1) the client gives informed consent;
(2) there is no interference with the lawyer's independence of professional judgment or
with the client-lawyer relationship; and
(3) information relating to representation of a client is protected as required by Rule 1.6.

19
(g) A lawyer who represents two or more clients shall not participate in making an aggregate
settlement of the claims of or against the clients, or in a criminal case an aggregated agreement
as to guilty or nolo contendere pleas, unless each client gives informed consent, in a writing
signed by the client. The lawyer's disclosure shall include the existence and nature of all the
claims or pleas involved and of the participation of each person in the settlement.
(h) A lawyer shall not:
(1) make an agreement prospectively limiting the lawyer's liability to a client for
malpractice unless the client is independently represented in making the agreement; or
(2) settle a claim or potential claim for such liability with an unrepresented client or
former client unless that person is advised in writing of the desirability of seeking and is given a
reasonable opportunity to seek the advice of independent legal counsel in connection therewith.
(i) A lawyer shall not acquire a proprietary interest in the cause of action or subject
matter of litigation the lawyer is conducting for a client, except that the lawyer may:
(1) acquire a lien authorized by law to secure the lawyer's fee or expenses; and
(2) contract with a client for a reasonable contingent fee in a civil case.
(j) A lawyer shall not have sexual relations with a client unless a consensual sexual relationship
existed between them when the client-lawyer relationship commenced.
(k) While lawyers are associated in a firm, a prohibition in the foregoing paragraphs (a) through
(i) that applies to any one of them shall apply to all of them.

Criminal Co-Defendants
 To protect client’s 6th Amendment right to counsel, you must notify the court as soon as a
conflict arises
 Must have full disclosure and then informed consent to continue representation or else
subject to discipline and client may be able to assert ineffective assistance of counsel on
appeal

Representing Co-Parties
 Assess likelihood of conflict
 Co-defendants: Similar risks as presented by co-defendants in criminal case except you
can waive cross claims and can agree on how to split liability before the suit is filed.
 Co-plaintiffs: Easier, but must assess the commonality of objectives and power balance
among all the plaintiffs
 Joint defense or joint representation agreement must be airtight re: sharing confidential
information, flow of information, and what happens if a conflict arises

Co-Parties in Transactions
 Representing parties in forming a business, making an investment, prosecuting a patent,
etc.
 How likely are the clients able to each articulate their own interests --- are there
imbalances of power among the clients that would interfere with your ability to represent
each equally?
 How likely is the transaction to be successful? How much information do the clients
have? How realistic are they being?

20
 What are the implications if the deal falls apart?

Key Terms for your Engagement Agreement b/w Co-Parties


 benefits and risks of joint representation
 effects on confidentiality and sharing information
 effects on privilege (waiving a/c priv among the parties)
 how you will deal with withdrawal should that arise

Class Actions
 Attorneys represent only the named class representative
 Rules for class certification mediate some of the conflicts issues -- insure commonality,
representativeness, etc.
 Factors:
o Common issues
o Separate representation impractical
o Extent of active judicial supervision Restatement (Third) of the Law Governing
Lawyers §128 cmt. d(iii) (2000).” ABA/BNA 51:301

Consenting to Conflicts
 Interests giving rise to the conflict
 Contingent, optional, and tactical considerations and alternative courses of action that
would be foreclosed or made less readily available by the conflict
 Effect of the representation or the process of obtaining other clients' informed consent
upon confidential information of the client
 Any material reservations that a disinterested lawyer might reasonably harbor about the
arrangement if such a lawyer were representing only the client being advised
 Consequences and effect of a future withdrawal of consent by the client, including, if
relevant, the fact that the lawyer would withdraw from representing all clients. 

Rule 1.9 Duties To Former Clients


(a) A lawyer who has formerly represented a client in a matter shall not thereafter represent
another person in the same or a substantially related matter in which that person's interests are
materially adverse to the interests of the former client unless the former client gives informed
consent, confirmed in writing.
(b) A lawyer shall not knowingly represent a person in the same or a substantially related matter
in which a firm with which the lawyer formerly was associated had previously represented a
client
(1) whose interests are materially adverse to that person; and
(2) about whom the lawyer had acquired information protected by Rules 1.6 and 1.9(c)
that is material to the matter;
unless the former client gives informed consent, confirmed in writing.
(c) A lawyer who has formerly represented a client in a matter or whose present or former firm
has formerly represented a client in a matter shall not thereafter:

21
(1) use information relating to the representation to the disadvantage of the former client
except as these Rules would permit or require with respect to a client, or when the information
has become generally known; or
(2) reveal information relating to the representation except as these Rules would permit
or require with respect to a client.

“May use info of former client w/o their consent if it is generally known”
 Generally known and public records are NOT the same thing
 Refers only to the USE of client info – not the disclosure of client info
 NOTE: MO does have 1 Supreme Court case that said there is a public record exception
to the duty of confidentiality (w/ regards to former clients)
o Only state in the union that has such an opinion
 BGF thinks opinion was wrongfully decided

1.9 Former Clients


 Does your client know they are “former” or do they have a reasonable expectation that
they are a current client due to a repeated pattern of engagement?
 Duty of loyalty is broader than duty of confidentiality or an attorney-client privilege
 Duty does not depent solely on whether there is confidential info
 Can’t just switch sides and oppose client even if there is no confidential info
 1.9 conflicts are consentable. Some under 1.7 are not.

When is someone a former client?


 When your client has fired you or if you have withdrawn
 When you have completed everything w/in the scope of the representation that you have
been hired to do
o It is when YOU are done – not when they are done. So if you have completed
your representation but client still owes you money then that client is still a
former client

In Re Anonymous
 Atty gave AB a referral for a divorce lawyer at her own firm after AB told atty that she
wanted to divorce her husband after they had an altercation where her husband told police
that she threatened to harm him. AB and her husband eventually made up though.
However, atty did not know this and a friend of AB’s ata social gathering – atty told that
friend about AB’s filing for divorce and about her husband’s accusation. Atty
encouraged AB’s friend to contact AB b/c the friend expressed concern for her. When
AB’s friend called, AB became upset about the revelation of information and she filed a
grievance against atty.
 Commission charged atty w/ violating 1.9 – court concluded that she did violate 1.9 by
improperly revealing info relating to the representation of a former client.
 This falls under 1.9 b/c it includes “a lawyer who has formerly represented a client in a
matter or whose present or former firm has formerly represented a client in a
matter.”  So b/c atty referred AB to an atty in her own firm this falls under 1.9

22
 Note: court doesn’t seem to care much that the atty’s disclosure of this info was done in
concern for AB – it was still improper.

In Re Carey
 MR 1.9 former representation – lawyer shall not represent another in “the same or a substantially
related matter”
 Defended in product liability class actions re: heater cores & mini van latches
 Carey then left and joined his own firm – Carey & Dannis
 now representing Beam suing Chrysler in brakes cases
 1.9 says can’t represent someone materially adverse. But is the
same or substantially related matter?
o Carey said no b/c he didn’t know anything about the
brakes. Only knew about heater cores & mini van latches –
Carey said it had to be identical facts for it to be the same
matter
 Pg. 432 – also describes the process that Carey went thru to determine if there was a conflict or
not
 says he did research, so why didn’t his research work?
 didn’t look for a neutral opinion b/c he did not recognize his own
bias.
 also don’t know how he researched so he might not have been very
good at it
 Court imposes discipline in this case – one of the few cases that court imposes discipline instead
of disqualification
 most conflicts of interest cases are handled by disqualification
 discipline cases are very rare
 NOTE: Same and substantially related test is not just about fact! It is what you would likely
have been able to learn in a prior representation that could be used against your former client
(even if you didn’t actually learn it!)

Rule 1.7 Rule 1.9(a)

Whose interests can create the conflict? Another current New Client v.
client, former Former Client
client, 3rd
person, or by a
personal interest
of the lawyer
How must the interest of the conflicting party affect the current Directly Adverse Materially Adverse
client’s interests? or Materially
Limited
What relationship between the two representations is necessary?  A concurrent  Current client’s
conflict of matter must be the
interest same or substantially
related matter as a

23
former client’s
matter
Is consent possible?  YES; if each  YES; if former
affected client client gives
gives informed informed consent,
consent, confirmed in
confirmed in writing.
writing

Rule 1.10 Imputation Of Conflicts Of Interest: General Rule


(a) While lawyers are associated in a firm, none of them shall knowingly represent a client when
any one of them practicing alone would be prohibited from doing so by Rules 1.7 or 1.9, unless
(1) the prohibition is based on a personal interest of the disqualified lawyer and does not
present a significant risk of materially limiting the representation of the client by the remaining
lawyers in the firm; or
(2) the prohibition is based upon Rule 1.9(a) or (b) and arises out of the disqualified
lawyer’s association with a prior firm, and
(i) the disqualified lawyer is timely screened from any participation in the matter
and is apportioned no part of the fee therefrom;
(ii) written notice is promptly given to any affected former client to enable the
former client to ascertain compliance with the provisions of this Rule, which shall include
a description of the screening procedures employed; a statement of the firm's and of the
screened lawyer's compliance with these Rules; a statement that review may be available
before a tribunal; and an agreement by the firm to respond promptly to any written
inquiries or objections by the former client about the screening procedures; and
(iii) certifications of compliance with these Rules and with the screening
procedures are provided to the former client by the screened lawyer and by a partner of
the firm, at reasonable intervals upon the former client's written request and upon
termination of the screening procedures.
(b) When a lawyer has terminated an association with a firm, the firm is not prohibited from
thereafter representing a person with interests materially adverse to those of a client represented
by the formerly associated lawyer and not currently represented by the firm, unless:
(1) the matter is the same or substantially related to that in which the formerly associated
lawyer represented the client; and
(2) any lawyer remaining in the firm has information protected by Rules 1.6 and 1.9(c)
that is material to the matter.
(c) A disqualification prescribed by this rule may be waived by the affected client under the
conditions stated in Rule 1.7.
(d) The disqualification of lawyers associated in a firm with former or current government
lawyers is governed by Rule 1.11.

Rule 1.11 Special Conflicts Of Interest For Former And Current Government Officers And
Employees
(a) Except as law may otherwise expressly permit, a lawyer who has formerly served as a public

24
officer or employee of the government:
(1) is subject to Rule 1.9(c); and
(2) shall not otherwise represent a client in connection with a matter in which the lawyer
participated personally and substantially as a public officer or employee, unless the appropriate
government agency gives its informed consent, confirmed in writing, to the representation.
(b) When a lawyer is disqualified from representation under paragraph (a), no lawyer in a firm
with which that lawyer is associated may knowingly undertake or continue representation in such
a matter unless:
(1) the disqualified lawyer is timely screened from any participation in the matter and is
apportioned no part of the fee therefrom; and
(2) written notice is promptly given to the appropriate government agency to enable it to
ascertain compliance with the provisions of this rule.
(c) Except as law may otherwise expressly permit, a lawyer having information that the lawyer
knows is confidential government information about a person acquired when the lawyer was a
public officer or employee, may not represent a private client whose interests are adverse to that
person in a matter in which the information could be used to the material disadvantage of that
person. As used in this Rule, the term "confidential government information" means information
that has been obtained under governmental authority and which, at the time this Rule is applied,
the government is prohibited by law from disclosing to the public or has a legal privilege not to
disclose and which is not otherwise available to the public. A firm with which that lawyer is
associated may undertake or continue representation in the matter only if the disqualified lawyer
is timely screened from any participation in the matter and is apportioned no part of the fee
therefrom.
(d) Except as law may otherwise expressly permit, a lawyer currently serving as a public officer
or employee:
(1) is subject to Rules 1.7 and 1.9; and
(2) shall not:
(i) participate in a matter in which the lawyer participated personally and
substantially while in private practice or nongovernmental employment, unless the
appropriate government agency gives its informed consent, confirmed in writing; or
(ii) negotiate for private employment with any person who is involved as a party
or as lawyer for a party in a matter in which the lawyer is participating personally and
substantially, except that a lawyer serving as a law clerk to a judge, other adjudicative
officer or arbitrator may negotiate for private employment as permitted by Rule 1.12(b)
and subject to the conditions stated in Rule 1.12(b).
(e) As used in this Rule, the term "matter" includes:
(1) any judicial or other proceeding, application, request for a ruling or other
determination, contract, claim, controversy, investigation, charge, accusation, arrest or other
particular matter involving a specific party or parties, and
(2) any other matter covered by the conflict of interest rules of the appropriate
government agency.

Rule 1.12 Former Judge, Arbitrator, Mediator Or Other Third-Party Neutral


(a) Except as stated in paragraph (d), a lawyer shall not represent anyone in connection with a
matter in which the lawyer participated personally and substantially as a judge or other
adjudicative officer or law clerk to such a person or as an arbitrator, mediator or other third-party

25
neutral, unless all parties to the proceeding give informed consent, confirmed in writing.
(b) A lawyer shall not negotiate for employment with any person who is involved as a party or as
lawyer for a party in a matter in which the lawyer is participating personally and substantially as
a judge or other adjudicative officer or as an arbitrator, mediator or other third-party neutral. A
lawyer serving as a law clerk to a judge or other adjudicative officer may negotiate for
employment with a party or lawyer involved in a matter in which the clerk is participating
personally and substantially, but only after the lawyer has notified the judge or other adjudicative
officer.
(c) If a lawyer is disqualified by paragraph (a), no lawyer in a firm with which that lawyer is
associated may knowingly undertake or continue representation in the matter unless:
(1) the disqualified lawyer is timely screened from any participation in the matter and is
apportioned no part of the fee therefrom; and
(2) written notice is promptly given to the parties and any appropriate tribunal to enable
them to ascertain compliance with the provisions of this rule.
(d) An arbitrator selected as a partisan of a party in a multimember arbitration panel is not
prohibited from subsequently representing that party.

Rule 1.13 Organization As Client


(a) A lawyer employed or retained by an organization represents the organization acting through
its duly authorized constituents.
(b) If a lawyer for an organization knows that an officer, employee or other person associated
with the organization is engaged in action, intends to act or refuses to act in a matter related to
the representation that is a violation of a legal obligation to the organization, or a violation of law
that reasonably might be imputed to the organization, and that is likely to result in substantial
injury to the organization, then the lawyer shall proceed as is reasonably necessary in the best
interest of the organization. Unless the lawyer reasonably believes that it is not necessary in the
best interest of the organization to do so, the lawyer shall refer the matter to higher authority in
the organization, including, if warranted by the circumstances to the highest authority that can
act on behalf of the organization as determined by applicable law.
(c) Except as provided in paragraph (d), if
(1) despite the lawyer's efforts in accordance with paragraph (b) the highest authority that
can act on behalf of the organization insists upon or fails to address in a timely and appropriate
manner an action, or a refusal to act, that is clearly a violation of law, and
(2) the lawyer reasonably believes that the violation is reasonably certain to result in
substantial injury to the organization, then the lawyer may reveal information relating to the
representation whether or not Rule 1.6 permits such disclosure, but only if and to the extent the
lawyer reasonably believes necessary to prevent substantial injury to the organization.
(d) Paragraph (c) shall not apply with respect to information relating to a lawyer's representation
of an organization to investigate an alleged violation of law, or to defend the organization or an
officer, employee or other constituent associated with the organization against a claim arising out
of an alleged violation of law.
(e) A lawyer who reasonably believes that he or she has been discharged because of the lawyer's
actions taken pursuant to paragraphs (b) or (c), or who withdraws under circumstances that
require or permit the lawyer to take action under either of those paragraphs, shall proceed as the
lawyer reasonably believes necessary to assure that the organization's highest authority is
informed of the lawyer's discharge or withdrawal.

26
(f) In dealing with an organization's directors, officers, employees, members, shareholders or
other constituents, a lawyer shall explain the identity of the client when the lawyer knows or
reasonably should know that the organization's interests are adverse to those of the constituents
with whom the lawyer is dealing.
(g) A lawyer representing an organization may also represent any of its directors, officers,
employees, members, shareholders or other constituents, subject to the provisions of Rule 1.7. If
the organization's consent to the dual representation is required by Rule 1.7, the consent shall be
given by an appropriate official of the organization other than the individual who is to be
represented, or by the shareholders

Navigating Rule 1.13(b):

1.   Do you REPRESENT THE ENTITY?


IF YES, IF NO, Rule 1.13 Does not create a compliance duty
then  
  You not only have no duty, but should not be providing legal advice to an entity that you do not represent
(See Rules 4.2 & 4.3).
 
2.   Is there a threat of LIKELY SUBSTANTIAL INJURY?
IF YES, IF NO, Rule 1.13 Does not create a compliance duty
then  
  Attorneys may but need not alert clients to risks of minor or remote harm (cmt. 4, Rule 1.13). They do not,
however, have a duty to counsel compliance to avoid these risks.
 
3. Is the injury TO THE ENTITY?
IF YES, IF NO, Rule 1.13 Does not create a compliance duty
then  
  Attorneys may not assist their clients in criminal or fraudulent conduct (Rule 1.2(d)). Other duties and
  exceptions might apply (e.g., Rule 1.6(b), 3.3, 4.1), but Rule 1.13 does not apply  (See cmt. 6 Rule 1.13).

4.   Is the injury the result of ACTS, INACTION, OR PLANS of


an officer, employee or other PERSON ASSOCIATED WITH THE ENTITY?
IF YES, IF NO, Rule 1.13 Does not create a compliance duty
then  
  You may have a duty under Rules 1.1 and 1.4 to counsel the entity regarding an external threat, but not
  under Rule 1.13 (i.e., this is not a compliance issue).
 
5.   Is the threat in a matter that is RELATED TO THE REPRESENTATION?
IF YES, IF NO, Rule 1.13 Does not create a compliance duty
then  
If the matter is outside the scope of your representation, you may but need not invite the client to expand
the scope but may not do so unilaterally. (See cmt. 5, Rule 2.1)
 
6. How confident are you about the threat (i.e., do you KNOW  of the  act, inaction, or plan that will result in
injury?)
IF YES, IF NO, Rule 1.13 Does not create a compliance duty

27
then  
If you only guess, or suspect, you may have a duty to investigate if the matter is one for which you
otherwise have a duty to provide advice or for which you represent the client.  Otherwise, you ordinarily
have no duty to investigate your own client. (See cmt. 5, rule 2.1)
 
7.   Does the act, inaction, or plan
VIOLATE A DUTY TO THE ENTITY or
IMPUTE A LEGAL VIOLATION TO THE ENTITY?
IF YES, IF NO, Rule 1.13 Does not create a compliance duty
then  
Only when the law is violated does the attorney’s duty attach.  If the actions or plans are legal, the attorney
should ordinarily accept entity decisions even if the actions are imprudent or unwise. (Cmt. 3, Rule 1.13).
 
Attorney has the duty to proceed as is REASONABLY NECESSARY
in the BEST INTERESTS OF THE ORGANIZATION.
 
Note that comment 4 to the rule identifies a number of factors relevant to  this plan: the seriousness of the conduct
and consequences, the motives and responsibility of the constituent and company policies.

Rule 1.14 Client With Diminished Capacity


(a) When a client's capacity to make adequately considered decisions in connection with a
representation is diminished, whether because of minority, mental impairment or for some other
reason, the lawyer shall, as far as reasonably possible, maintain a normal client-lawyer
relationship with the client.
(b) When the lawyer reasonably believes that the client has diminished capacity, is at risk of
substantial physical, financial or other harm unless action is taken and cannot adequately act in
the client's own interest, the lawyer may take reasonably necessary protective action, including
consulting with individuals or entities that have the ability to take action to protect the client and,
in appropriate cases, seeking the appointment of a guardian ad litem, conservator or guardian.
(c) Information relating to the representation of a client with diminished capacity is protected by
Rule 1.6. When taking protective action pursuant to paragraph (b), the lawyer is impliedly
authorized under Rule 1.6(a) to reveal information about the client, but only to the extent
reasonably necessary to protect the client's interests.

Rule 1.15 Safekeeping Property


(a) A lawyer shall hold property of clients or third persons that is in a lawyer's possession in
connection with a representation separate from the lawyer's own property. Funds shall be kept in
a separate account maintained in the state where the lawyer's office is situated, or elsewhere with
the consent of the client or third person. Other property shall be identified as such and
appropriately safeguarded. Complete records of such account funds and other property shall be
kept by the lawyer and shall be preserved for a period of [five years] after termination of the
representation.
(b) A lawyer may deposit the lawyer's own funds in a client trust account for the sole purpose of
paying bank service charges on that account, but only in an amount necessary for that purpose.
(c) A lawyer shall deposit into a client trust account legal fees and expenses that have been paid
in advance, to be withdrawn by the lawyer only as fees are earned or expenses incurred.

28
(d) Upon receiving funds or other property in which a client or third person has an interest, a
lawyer shall promptly notify the client or third person. Except as stated in this rule or otherwise
permitted by law or by agreement with the client, a lawyer shall promptly deliver to the client or
third person any funds or other property that the client or third person is entitled to receive and,
upon request by the client or third person, shall promptly render a full accounting regarding such
property.
(e) When in the course of representation a lawyer is in possession of property in which two or
more persons (one of whom may be the lawyer) claim interests, the property shall be kept
separate by the lawyer until the dispute is resolved. The lawyer shall promptly distribute all
portions of the property as to which the interests are not in dispute.
Trust Accounting
 One trust account for all clients but separate “accounting” for each client
 IOLTA foundation for the interest on the trust account (lawyer pays the interest)
 Can delegate duty but can’t delegate the responsibility
 Don’t co-mingle funds
 Don’t distribute if there is a dispute

Rule 1.16 Declining Or Terminating Representation


(a) Except as stated in paragraph (c), a lawyer shall not represent a client or, where
representation has commenced, shall withdraw from the representation of a client if:
(1) the representation will result in violation of the rules of professional conduct or other
law;
(2) the lawyer's physical or mental condition materially impairs the lawyer's ability to
represent the client; or
(3) the lawyer is discharged.
(b) Except as stated in paragraph (c), a lawyer may withdraw from representing a client if:
(1) withdrawal can be accomplished without material adverse effect on the interests of
the client;
(2) the client persists in a course of action involving the lawyer's services that the lawyer
reasonably believes is criminal or fraudulent;
(3) the client has used the lawyer's services to perpetrate a crime or fraud;
(4) the client insists upon taking action that the lawyer considers repugnant or with which
the lawyer has a fundamental disagreement;
(5) the client fails substantially to fulfill an obligation to the lawyer regarding the
lawyer's services and has been given reasonable warning that the lawyer will withdraw unless the
obligation is fulfilled;
(6) the representation will result in an unreasonable financial burden on the lawyer or has
been rendered unreasonably difficult by the client; or
(7) other good cause for withdrawal exists.
(c) A lawyer must comply with applicable law requiring notice to or permission of a tribunal
when terminating a representation. When ordered to do so by a tribunal, a lawyer shall continue
representation notwithstanding good cause for terminating the representation.
(d) Upon termination of representation, a lawyer shall take steps to the extent reasonably
practicable to protect a client's interests, such as giving reasonable notice to the client, allowing
time for employment of other counsel, surrendering papers and property to which the client is

29
entitled and refunding any advance payment of fee or expense that has not been earned or
incurred. The lawyer may retain papers relating to the client to the extent permitted by other law.

*Swift, Currie, McGhee & Heirs v. Henry


 Question: Does a document created by an atty in the course of representing a client
belong to the atty or the client?
 Minority of courts: document belongs to atty who prepared it, unless doc. is
sought by the client in connection w/ a lawsuit against the atty. These
jurisdictions often employ a work product analysis and take the position that an
atty can raise the work product privilege and if that applies then the client cannot
compel the atty to disclose the document
 Majority of courts: document created by an atty belongs to the client who
retained him. However, good cause to refuse discovery would arise if disclosure
would violate an atty’s duty to a third party. Good cause might also be shown in
the doc. assesses the client himself, or includes “tentative preliminary impressions
of the lgal or factual issues presented in the representation, recorded primarily for
the purpose of giving internal direction to facilitate performance of the legal
services entailed in that representation.”
 Ct adopts the majority approach and holds that the file of the client belongs to the client
and atty cannot refuse to turn over file absent good cause. Why this is here is because the
test for turning over the file is that you must put the client in a position that she can
reasonably continue representation with another attorney. (sounds similar to subsection
(d)

1.16 Can Be Read As:


 A set of guidelines for rejecting clients
 A tool for client communication
 A detailed but not exclusive list of restrictions on your ability to withdraw from a
representation

Ending Joint Representation


 Settlement of joint claims carries special responsibilities under Rule 1.8(g)
 Clients have the right to withdraw their consent to joint representation at any time, which
requires the attorney to then withdraw from the representation (1.16(a))
 If an attorney withdraws from a joint representation, he or she cannot ordinarily choose to
continue to represent one or the other of the clients. All or none.

State v. Gonzales
 Atty permitted to withdraw under 1.16 because she had knowledge that former client was
going to commit perjury Ms. Gonzales’s case and this would materially limit her ability
to cross-examine the witness (1.7(a)(2) would create a conflict). This is really a better
case for explaining disclosure under the crime/fraud exceptions under 1.6 and 3.3 candor
to the tribunal (disclosing clients intent to commit perjury).

Rule 1.18 Duties To Prospective Client

30
(a) A person who consults with a lawyer about the possibility of forming a client-lawyer
relationship with respect to a matter is a prospective client.
(b) Even when no client-lawyer relationship ensues, a lawyer who has learned information from
a prospective client shall not use or reveal that information, except as Rule 1.9 would permit with
respect to information of a former client.
(c) A lawyer subject to paragraph (b) shall not represent a client with interests materially
adverse to those of a prospective client in the same or a substantially related matter if the lawyer
received information from the prospective client that could be significantly harmful to that
person in the matter, except as provided in paragraph (d). If a lawyer is disqualified from
representation under this paragraph, no lawyer in a firm with which that lawyer is associated may
knowingly undertake or continue representation in such a matter, except as provided in
paragraph (d).
(d) When the lawyer has received disqualifying information as defined in paragraph (c),
representation is permissible if:
(1) both the affected client and the prospective client have given informed consent,
confirmed in writing, or:
(2) the lawyer who received the information took reasonable measures to avoid
exposure to more disqualifying information than was reasonably necessary to determine whether
to represent the prospective client; and
(i) the disqualified lawyer is timely screened from any participation in the matter and is
apportioned no part of the fee therefrom; and
(ii) written notice is promptly given to the prospective client.

In Re AnonymousRecall that AB talked to lawyer about divorce in order to get a referral.


Learning this information this way did not mean lawyer had duty of confidentiality to AB
because AB did not expect lawyer to represent her. If AB’s communication w/ atty had ended w/
that phone call, revelation of the info at issue would not have been a violation of atty’s ethical
duties. Comment 2 I believe states that communicating info to lawyer does not make person
prospective client when no expectation of representation or if lawyer doesn’t indicate at any time
that they possibly will represent. The info at issue however, was disclosed to atty not long b4 the
second call in which AB asked for an atty referral and atty referred AB to an atty from her own
firm. At that point, if not before, AB became a prospective client under 1.18. She then became a
client under 1.9 b/c 1.9 states that “a lawyer who has formerly represented a client in a matter or
whose present or former firm has formerly represented a client in a matter.”  therefore,
atty was in violation of 1.9

Full Disclosure & Waiver of Prospective Clients


 Must Have:
o Affirmative revelation
o By the attorney of
o All the facts, legal implications, possible effects, and other circumstances relating
to the proposed representation

Caveats to Full Disclosure:


 A client’s mere knowledge of the existence of his attorney’s other representation does not
alone constitute full disclosure (Dresser)

31
 Unlikely that a prospective waiver which did not identify either the potential opposing
party or at least a class of potentially conflicting clients would survive scrutiny
 Just b/c client consents, doesn’t mean that you don’t have to do the DAML analysis under
1.7
 What would disinterested attorney think?

COUNSELOR

Rule 2.1 Advisor


In representing a client, a lawyer shall exercise independent professional judgment and render
candid advice. In rendering advice, a lawyer may refer not only to law but to other considerations
such as moral, economic, social and political factors, that may be relevant to the client's
situation.

Rule 2.3 Evaluation For Use By Third Persons


(a) A lawyer may provide an evaluation of a matter affecting a client for the use of someone
other than the client if the lawyer reasonably believes that making the evaluation is compatible
with other aspects of the lawyer's relationship with the client.
(b) When the lawyer knows or reasonably should know that the evaluation is likely to affect the
client's interests materially and adversely, the lawyer shall not provide the evaluation unless the
client gives informed consent.
(c) Except as disclosure is authorized in connection with a report of an evaluation, information
relating to the evaluation is otherwise protected by Rule 1.6.

ADVOCATE

Rule 3.1 Meritorious Claims And Contentions


A lawyer shall not bring or defend a proceeding, or assert or controvert an issue therein, unless
there is a basis in law and fact for doing so that is not frivolous, which includes a good faith
argument for an extension, modification or reversal of existing law. A lawyer for the defendant
in a criminal proceeding, or the respondent in a proceeding that could result in incarceration,
may nevertheless so defend the proceeding as to require that every element of the case be
established.

Rule 3.3 Candor Toward The Tribunal


(a) A lawyer shall not knowingly:
(1) make a false statement of fact or law to a tribunal or fail to correct a false statement of
material fact or law previously made to the tribunal by the lawyer;
(2) fail to disclose to the tribunal legal authority in the controlling jurisdiction known to
the lawyer to be directly adverse to the position of the client and not disclosed by opposing
counsel; or
(3) offer evidence that the lawyer knows to be false. If a lawyer, the lawyer’s client, or a

32
witness called by the lawyer, has offered material evidence and the lawyer comes to know of its
falsity, the lawyer shall take reasonable remedial measures, including, if necessary, disclosure to
the tribunal. A lawyer may refuse to offer evidence, other than the testimony of a defendant in a
criminal matter, that the lawyer reasonably believes is false.
(b) A lawyer who represents a client in an adjudicative proceeding and who knows that a person
intends to engage, is engaging or has engaged in criminal or fraudulent conduct related to the
proceeding shall take reasonable remedial measures, including, if necessary, disclosure to the
tribunal.
(c) The duties stated in paragraphs (a) and (b) continue to the conclusion of the proceeding, and
apply even if compliance requires disclosure of information otherwise protected by Rule 1.6.
(d) In an ex parte proceeding, a lawyer shall inform the tribunal of all material facts known to the
lawyer that will enable the tribunal to make an informed decision, whether or not the facts are
adverse.

Note: A key element to both 3.3(a)(1) and (3) is materiality. 

3.3(a)(3) does not permit an atty to refuse the client’s demand to testify, even if the atty
reasonably believes the testimony will be false, if the client is a criminal defendant. However,
the rule does NOT permit an atty to knowingly submit false evidence, even in a criminal case

Nix v. Whitseide  atty knew that his client would testify falsely b/c he announced his intention
to do so. Atty was able to dissuade his client from perjury and, thus, was able to call him to
testify. Here, the defendant’s 6th amendment right of a criminal defendant to assistance of
counsel is not violated when atty refuses to cooperate w/ the defendant in presenting perjured
testimony at his trial.

United States v. Long  Long represented defendant and in trial Long announced he might have
to withdrawal if Long testified b/c he believed the testimony would be perjury. Defendant then
agreed not to testify. But case differs from Whiteside in 3 diff. respects:
 In Whiteside, a finding was made that he would have testified falsely had he given the
testimony he initially wanted to give – but here, no such finding has been made. Long did
not have a “firm factual basis” for believing defendant would testify falsely and needs to
be decided in an evidentiary hearing
 2nd – def. testified and was “restricted” or restrained only from testifying falsely; but here,
def. did not testify at all, so it is impossible to determine whether def. was restrained by
his lawyer from giving truthful testimony (can only be determined in evidentiary hearing
 3rd – defesne atty did not reveal his believe about his client’s anticipated testimony;
whereas here he did, which creates a “significant risk of unfair prejudice” to def. Trial ct.
reduced these prejudicies though and it is necessary to establish if def. waived his
constitutional right to testify at an evidentiary hearing.

Key Questions:
 Rule 3.3(a)(3) and 3.3(b) both impose a duty to take reasonable remedial measure.  3.3(a)
(3) requires that the attorney address “false material evidence” and 3.3(b) requires
reasonable remedial measures to address “criminal or fraudulent conduct related to the
proceeding”.  How do these two compare?

33
 Isn’t the duty under Rule 3.3(a)(3) just a subset of the duty imposed by Rule 3.3(b)?  Isn’t
submitting false material evidence always a crime or fraud on the tribunal?
 Rule 1.16, 1.6, and 3.3 all address situations in which a client might be engaged in fraud
or perjury.  Compare these carefully.  What must believe or know about your client’s
intended wrongdoing in order to withdraw? To disclose to third parties?  To disclose to
the court? In United States v. Long, when the attorney reasonably believed his client was
going to lie, why wasn’t withdrawal the best solution?
 One of the keys to all of these questions is whether the attorney “knows” – how does an
attorney know what another person intends?  How does at attorney “know” that evidence
is false?  What if he only suspects?  In Nix, would Robinson “know” that Whiteside was
lying with he hadn’t admitted that he thought he needed to say that there was a gun?

Rule 3.7 Lawyer As Witness


(a) A lawyer shall not act as advocate at a trial in which the lawyer is likely to be a necessary
witness unless:
(1) the testimony relates to an uncontested issue;
(2) the testimony relates to the nature and value of legal services rendered in the case; or
(3) disqualification of the lawyer would work substantial hardship on the client.
(b) A lawyer may act as advocate in a trial in which another lawyer in the lawyer's firm is likely
to be called as a witness unless precluded from doing so by Rule 1.7 or Rule 1.9.

TRANSACTIONS WITH PERSONS OTHER THAN CLIENTS

Rule fr Truthfulness In Statements To Others


In the course of representing a client a lawyer shall not knowingly:
(a) make a false statement of material fact or law to a third person; or
(b) fail to disclose a material fact to a third person when disclosure is necessary to avoid
assisting a criminal or fraudulent act by a client, unless disclosure is prohibited by Rule 1.6.

Rule 4.2 Communication With Person Represented By Counsel


In representing a client, a lawyer shall not communicate about the subject of the representation
with a person the lawyer knows to be represented by another lawyer in the matter, unless the
lawyer has the consent of the other lawyer or is authorized to do so by law or a court order.

Rule 4.3 Dealing With Unrepresented Person


In dealing on behalf of a client with a person who is not represented by counsel, a lawyer shall
not state or imply that the lawyer is disinterested. When the lawyer knows or reasonably should
know that the unrepresented person misunderstands the lawyer’s role in the matter, the lawyer
shall make reasonable efforts to correct the misunderstanding. The lawyer shall not give legal
advice to an unrepresented person, other than the advice to secure counsel, if the lawyer knows
or reasonably should know that the interests of such a person are or have a reasonable possibility
of being in conflict with the interests of the client.

LAW FIRMS AND ASSOCIATIONS

Rule 5.1 Responsibilities Of Partners, Managers, And Supervisory Lawyers

34
(a) A partner in a law firm, and a lawyer who individually or together with other lawyers
possesses comparable managerial authority in a law firm, shall make reasonable efforts to ensure
that the firm has in effect measures giving reasonable assurance that all lawyers in the firm
conform to the Rules of Professional Conduct.
(b) A lawyer having direct supervisory authority over another lawyer shall make reasonable
efforts to ensure that the other lawyer conforms to the Rules of Professional Conduct.
(c) A lawyer shall be responsible for another lawyer's violation of the Rules of Professional
Conduct if:
(1) the lawyer orders or, with knowledge of the specific conduct, ratifies the conduct
involved; or
(2) the lawyer is a partner or has comparable managerial authority in the law firm in
which the other lawyer practices, or has direct supervisory authority over the other lawyer, and
knows of the conduct at a time when its consequences can be avoided or mitigated but fails to
take reasonable remedial action.

Rule 5.2 Responsibilities Of A Subordinate Lawyer


(a) A lawyer is bound by the Rules of Professional Conduct notwithstanding that the lawyer
acted at the direction of another person.
(b) A subordinate lawyer does not violate the Rules of Professional Conduct if that lawyer acts in
accordance with a supervisory lawyer's reasonable resolution of an arguable question of
professional duty.

Rule 5.3 Responsibilities Regarding Nonlawyer Assistance


With respect to a nonlawyer employed or retained by or associated with a lawyer:
(a) a partner, and a lawyer who individually or together with other lawyers possesses comparable
managerial authority in a law firm shall make reasonable efforts to ensure that the firm has in
effect measures giving reasonable assurance that the person's conduct is compatible with the
professional obligations of the lawyer;
(b) a lawyer having direct supervisory authority over the nonlawyer shall make reasonable
efforts to ensure that the person's conduct is compatible with the professional obligations of the
lawyer; and
(c) a lawyer shall be responsible for conduct of such a person that would be a violation of the
Rules of Professional Conduct if engaged in by a lawyer if:
(1) the lawyer orders or, with the knowledge of the specific conduct, ratifies the conduct
involved; or
(2) the lawyer is a partner or has comparable managerial authority in the law firm in
which the person is employed, or has direct supervisory authority over the person, and knows of
the conduct at a time when its consequences can be avoided or mitigated but fails to take
reasonable remedial action.

Rule 5.4 Professional Independence Of A Lawyer


(a) A lawyer or law firm shall not share legal fees with a nonlawyer, except that:
(1) an agreement by a lawyer with the lawyer's firm, partner, or associate may provide for the
payment of money, over a reasonable period of time after the lawyer's death, to the lawyer's
estate or to one or more specified persons;
(2) a lawyer who purchases the practice of a deceased, disabled, or disappeared lawyer may,

35
pursuant to the provisions of Rule 1.17, pay to the estate or other representative of that lawyer
the agreed-upon purchase price;
(3) a lawyer or law firm may include nonlawyer employees in a compensation or retirement plan,
even though the plan is based in whole or in part on a profit-sharing arrangement; and
(4) a lawyer may share court-awarded legal fees with a nonprofit organization that employed,
retained or recommended employment of the lawyer in the matter.
(b) A lawyer shall not form a partnership with a nonlawyer if any of the activities of the
partnership consist of the practice of law.
(c) A lawyer shall not permit a person who recommends, employs, or pays the lawyer to render
legal services for another to direct or regulate the lawyer's professional judgment in rendering
such legal services.
(d) A lawyer shall not practice with or in the form of a professional corporation or association
authorized to practice law for a profit, if:
(1) a nonlawyer owns any interest therein, except that a fiduciary representative of the estate of a
lawyer may hold the stock or interest of the lawyer for a reasonable time during administration;
(2) a nonlawyer is a corporate director or officer thereof or occupies the position of similar
responsibility in any form of association other than a corporation ; or
(3) a nonlawyer has the right to direct or control the professional judgment of a lawyer.

Rule 5.5 Unauthorized Practice Of Law; Multijurisdictional Practice Of Law


(a) A lawyer shall not practice law in a jurisdiction in violation of the regulation of the legal
profession in that jurisdiction, or assist another in doing so.
(b) A lawyer who is not admitted to practice in this jurisdiction shall not:
(1) except as authorized by these Rules or other law, establish an office or other
systematic and continuous presence in this jurisdiction for the practice of law; or
(2) hold out to the public or otherwise represent that the lawyer is admitted to practice
law in this jurisdiction.
(c) A lawyer admitted in another United States jurisdiction, and not disbarred or suspended from
practice in any jurisdiction, may provide legal services on a temporary basis in this jurisdiction
that:
(1) are undertaken in association with a lawyer who is admitted to practice in this
jurisdiction and who actively participates in the matter;
(2) are in or reasonably related to a pending or potential proceeding before a tribunal in
this or another jurisdiction, if the lawyer, or a person the lawyer is assisting, is authorized by law
or order to appear in such proceeding or reasonably expects to be so authorized;
(3) are in or reasonably related to a pending or potential arbitration, mediation, or other
alternative dispute resolution proceeding in this or another jurisdiction, if the services arise out of
or are reasonably related to the lawyer’s practice in a jurisdiction in which the lawyer is admitted
to practice and are not services for which the forum requires pro hac vice admission; or
(4) are not within paragraphs (c)(2) or (c)(3) and arise out of or are reasonably related to
the lawyer’s practice in a jurisdiction in which the lawyer is admitted to practice.
(d) A lawyer admitted in another United States jurisdiction or in a foreign jurisdiction, and not
disbarred or suspended from practice in any jurisdiction or the equivalent thereof, may provide
legal services through an office or other systematic and continuous presence in this jurisdiction
that :

36
(1) are provided to the lawyer’s employer or its organizational affiliates; are not services
for which the forum requires pro hac vice admission; and, when performed by a foreign lawyer
and requires advice on the law of this or another jurisdiction or of the United States, such advice
shall be based upon the advice of a lawyer who is duly licensed and authorized by the
jurisdiction to provide such advice; or
(2) are services that the lawyer is authorized by federal or other law or rule to provide in
this jurisdiction.
(e) For purposes of paragraph (d), the foreign lawyer must be a member in good standing of a
recognized legal profession in a foreign jurisdiction, the members of which are admitted to
practice as lawyers or counselors at law or the equivalent, and are subject to effective regulation
and discipline by a duly constituted professional body or a public authority.
State Adoption of Rule 5.5
 13 states have adopted identical versions (including AK, IL, Indiana, Iowa, and
Nebraska)
 31 states have adopted versions w/ variations (including MO, TN, KY, and OK)
 Only Hawaii, KS, Mississippi, Montana, NY, TX, and West Virginia have not adopted
some new version

What is the unauthorized Practice of Law (UPL):


 UPL is a territorial restriction
 Non-lawyers practicing law
 Lawyers practicing where they aren’t licensed or assisting non-lawyers in practicing law

Tests that Courts have used for UPL:


 Incidental Test – a layman may perform whatever legal services are incidental or
ancillary to established practices in his/her business
 Simple-Complex Test – if difficult, complex tasks are involved, only a lawyer can do
them. But if the tasks are simple, a layperson can do them
 Legal Skill & Knowledge Test – if no more than ordinary business intelligence is
required, a layperson can take action. If the activity is peculiar to one trained and
experienced in the law, then it’s the practice of law and a lawyer must act.

ABA Factors for Definition of Practice of Law


 Does the non-lawyer activity pose a serious risk to the consumer’s life, health, safety, or
economic well-being?
 Do potential customers of law-related non-lawyer services have the knowledge needed to
properly evaluate the qualifications of non-lawyers offering the services?
 Do the actual benefits of regulation likely to the public outweigh any likely negative
consequences of regulation?

What type of “Presence” are you engaging in?


 Systematic & Continuous Presence
 Open an Office
 Target Ads

37
 Ongoing physical presence
 Multiple Clients, regular physical presence
 Long Time Client, single matter
 Temporary practice
 Expert consulting; limited role

In Re Trester
 Trester admitted to KS bar but never passed California bar; however, Trester practiced
law in Cali for 40 years. Had a law office there and was advertised as “Law Offices of
Irwin Trester.”
 Friedman Bag Company, a Cali company, hired Trester. Trester never told them that he
was not licensed to practice in Cali. Friedman then found out and sued him alleging legal
malpractice and fraud. Trester then charged w/ UPL & & (later reduced to 3) counts of
theft.
 Hearing panel found Trester guilty of 5.5(a) and 8.4(b) & (c).
 8.4(b): found Trester in violation of committing “a criminal act that reflects adversely on
the lawyer’s honest, trustworthiness or fitness as a lawyer.”  Panel found that Trester’s
3 convictions of theft and 1 conviction of UPL are crimes that reflect directly on his
honesty and trustworthiness
 8.4(c): found Trester in violation of engaging “in conduct involving dishonesty, fraud,
deceit, or misrepresentation.”  Panel found that he engaged in dishonest conduct when
he held himself out as an atty in Cali.
 Trester tried to argue that his misconduct was not motivated by dishonest,
fraud, or deceit, but rather by the “mistaken belief” that he could hold
himself out as an atty b/c he was admitted to practice in KS and had been
admitted to practice law before the United States S.C., and the 9th and 10th
Circuit Court of Appeals. Trester said he ha a “plausible explanation” b/c he
solicited input from a Los Angeles atty who was head of the ethics division of
the Cali Bar and said that the services he provide were consistent w/ the advice
given by the LA guy. Furthermore, Trester said he never lied to
his clients by telling them he was admitted to the Cali bar and was limited to
federal practice. However, Trester’s “good faith” defense didn’t work b/c even
though he didn’t lie, he didn’t disclose either. Ct. said atty’s have an
independent duty to assess law and the underlying conduct here was serious,
ongoing dishonesty involving theft.

Defenses that DON’T work:


• Good faith –Trester
• Exclusively Federal Practice
 Huge split among states and circuits on this issue.
 Missouri takes a very conservative approach.
 The line again appears to be opening an office and holding oneself out in the state.

Consequences of Cross-Border Unauthorized Practice


 Civil Liability

38
o Tort
o Consumer Protection Statutes
o UPL statutes
 Criminal Convictions

What do you do if you want to practice in another state w/o risk?


 Get licensed (costly ways)
o Regular admission
o Admission by motion (reciprocity)
o Limited Admission
 In house counsel
 MO has registration statute for this
o Pro Hac Vice Admission

Where can you practice?


 Have to know your own State’s law and the law of any jurisdiction in which you will be
delivering legal services.

Remember: generally there is a difference in practicing the law of another state and
practicing law IN another state – only the latter is prohibited.

Rule 5.6 Restrictions On Right To Practice


A lawyer shall not participate in offering or making:
(a) a partnership, shareholders, operating, employment, or other similar type of
agreement that restricts the right of a lawyer to practice after termination of the relationship,
except an agreement concerning benefits upon retirement; or
(b) an agreement in which a restriction on the lawyer's right to practice is part of the
settlement of a client controversy.

Rule 5.7 Responsibilities Regarding Law-Related Services


(a) A lawyer shall be subject to the Rules of Professional Conduct with respect to the provision
of law-related services, as defined in paragraph (b), if the law-related services are provided:
(1) by the lawyer in circumstances that are not distinct from the lawyer's provision of
legal services to clients; or
(2) in other circumstances by an entity controlled by the lawyer individually or with
others if the lawyer fails to take reasonable measures to assure that a person obtaining the law-
related services knows that the services are not legal services and that the protections of the
client-lawyer relationship do not exist.
(b) The term "law-related services" denotes services that might reasonably be performed in
conjunction with and in substance are related to the provision of legal services, and that are not
prohibited as unauthorized practice of law when provided by a nonlawyer.

PUBLIC SERVICE

Rule 6.2 Accepting Appointments

39
A lawyer shall not seek to avoid appointment by a tribunal to represent a person except for good
cause, such as:
(a) representing the client is likely to result in violation of the Rules of Professional
Conduct or other law;
(b) representing the client is likely to result in an unreasonable financial burden on the
lawyer; or
(c) the client or the cause is so repugnant to the lawyer as to be likely to impair the client-
lawyer relationship or the lawyer's ability to represent the client.

Rule 6.3 Membership In Legal Services Organization


A lawyer may serve as a director, officer or member of a legal services organization, apart from
the law firm in which the lawyer practices, notwithstanding that the organization serves persons
having interests adverse to a client of the lawyer. The lawyer shall not knowingly participate in a
decision or action of the organization:
(a) if participating in the decision or action would be incompatible with the lawyer's
obligations to a client under Rule 1.7; or
(b) where the decision or action could have a material adverse effect on the
representation of a client of the organization whose interests are adverse to a client of the lawyer.

Rule 6.4 Law Reform Activities Affecting Client Interests


A lawyer may serve as a director, officer or member of an organization involved in reform of the
law or its administration notwithstanding that the reform may affect the interests of a client of the
lawyer. When the lawyer knows that the interests of a client may be materially benefitted by a
decision in which the lawyer participates, the lawyer shall disclose that fact but need not identify
the client.

Rule 6.5 Nonprofit And Court-Annexed Limited Legal Services Programs


(a) A lawyer who, under the auspices of a program sponsored by a nonprofit organization or
court, provides short-term limited legal services to a client without expectation by either the
lawyer or the client that the lawyer will provide continuing representation in the matter:
(1) is subject to Rules 1.7 and 1.9(a) only if the lawyer knows that the representation of
the client involves a conflict of interest; and
(2) is subject to Rule 1.10 only if the lawyer knows that another lawyer associated with
the lawyer in a law firm is disqualified by Rule 1.7 or 1.9(a) with respect to the matter.
(b) Except as provided in paragraph (a)(2), Rule 1.10 is inapplicable to a representation governed
by this Rule.

MAINTAINING THE INTEGRITY OF THE PROFESSION

Rule 8.3 Reporting Professional Misconduct


(a) A lawyer who knows that another lawyer has committed a violation of the Rules of
Professional Conduct that raises a substantial question as to that lawyer's honesty,
trustworthiness or fitness as a lawyer in other respects, shall inform the appropriate professional
authority.

40
(b) A lawyer who knows that a judge has committed a violation of applicable rules of judicial
conduct that raises a substantial question as to the judge's fitness for office shall inform the
appropriate authority.
(c) This Rule does not require disclosure of information otherwise protected by Rule 1.6 or
information gained by a lawyer or judge while participating in an approved lawyers assistance
program.

Note: MO’s 8.3 Requirement of reporting is not triggered by a single act of simple negligence

In Re Riehlmann
 Blood DNA case; Riehlmann’s friend, also an atty, told Riehlmann that he was dying of
colon cancer and in the same convo told him that he had suppressed exculpatory blood
evidence in a criminal case he prosecuted. Riehlmann was surprised and shocked by his
friends revelation and urged him to remedy the situation. However, neither of the men
remedied the situation
 5 yrs after his friends death, the defendant’s lawyers discovered a crime lab report and the
blood-stained physical evidence that had never been disclosed to defense counsel – the
crime lab report indicated that the robber had “Type B” blood but defendant had “type O”
Reihlmann heard of this evidence and then executed an affidavit saying “the late Gerry
Deegan said to me that he had intentionally suppressed blood evidence in the armed
robbery trial of John Thompson that in some way exculpated the defendant.”
 Riehlmann said he didn’t tell sooner b/c it was a difficult time in his life – his friend was
dying; he had left his wife w/ 3 kids; was under the care of a psychiatrist; and taking
antidepressants. Also went open-heart surgery and was distracted.
 To be required to report under 8.3, knowledge does not mean absolute certainty.
Knowledge requires more than a mere suspicion of ethical misconduct – court holds that
a lawyer will be found to have knowledge of reportable misconduct, and thus reporting is
required, where the supporting evidence is such that a reasonable lawyer under the
circumstances would form a firm belief that the conduct in question had more likely than
not occurred.
 When to report –Reporting should be made promptly
 Riehlmann should have promptly reported the misconduct to the disciplinary authorities –
b/c he didn’t, Riehlmann was publicly reprimanded

Rule 8.4 Misconduct (def. of misconduct is broad but not unconstitutionally broad)
It is professional misconduct for a lawyer to:
(a) violate or attempt to violate the Rules of Professional Conduct, knowingly assist or
induce another to do so, or do so through the acts of another;
(b) commit a criminal act that reflects adversely on the lawyer's honesty, trustworthiness
or fitness as a lawyer in other respects;
(c) engage in conduct involving dishonesty, fraud, deceit or misrepresentation;
(d) engage in conduct that is prejudicial to the administration of justice;
(e) state or imply an ability to influence improperly a government agency or official or to
achieve results by means that violate the Rules of Professional Conduct or other law; or
(f) knowingly assist a judge or judicial officer in conduct that is a violation of applicable
rules of judicial conduct or other law.

41
Rule 8.4(b) criminal act - "commit a criminal act" - a charge and conviction are not
necessary

Key interpretations of Rule 8.4


 Misconduct can be based on conduct not otherwise expressly governed by other
disciplinary rules, conduct not otherwise prohibited by law, and conduct while in one's
private capacity.
 Burden of Proof is on the state in discipline but on the applicant in admission
 Courts have generally upheld the broad and open-textured language of rule 8.4 as
constitutional
 Discipline is not punishment, but protection of the public, so double jeopardy does not
apply.

See Trester under 5.5!!

QUESTIONS & HYPOTETHICALS

Question 1
Which of the following best describes the scope of the information that is protected by the
attorney's duty of confidentiality under ABA Model Rule of Professional Conduct 1.6?

A. Information that would be significantly harmful to the client


B. Information relating to the representation
C. Communications between attorney and client in confidence for the purposes of seeking
legal advice
D. Materials prepared in anticipation of litigation
 
Question 2
Which of the following disclosures about a client would violate ABA Model Rule of
Professional Conduct 1.6 if the attorney did not have the client's consent to disclose the
information?

A. Disclosing the amount of an unpaid bill for legal services in a suit against the client to
recover those fees.
B. Without revealing the client's identity or any other fact not contained in a public record,
telling a "war story" to friends at a party about the client's testimony at the trial you are in
the middle of.
C. Disclosing the client's fraudulent statements on an securities statement that you helped
prepare in order to prevent individuals from relying on that statement and incurring
substantial financial loss.
D. Responding to a court's order that you disclose the client's statements to you regarding
plans for a crime that the client later carried out.
 

42
Question 3
Which of the following best defines the information that is protected by the evidentiary attorney-
client privilege?
 
A. Information relating to the representation
B. Information that would be significantly harmful to the client.
C. Communication between attorney and client in confidence for the purposes of seeking
legal advice.
D. Materials prepared in anticipation of litigation.
 

Question 4
You are representing a defendant-parent in a child abuse action in which your client is charged
with child neglect. The petition alleges that your client passed out from drinking and left his five-
year-old child without supervision. The child wandered off in the evening and was found blocks
away at a dangerous intersection. If requested to produce any of the following items of
information, for which of these would you properly raise an objection based on the attorney-
client privilege?
A. Your client's email to you describing what happened on the evening in question.
B. A witness statement you obtained from someone present in your client's home during the
evening in question.
C. Your client's driving record, which your client obtained from the sheriff's office and gave
to you, which shows a pattern of arrests for driving while intoxicated.
D. Your notes of the statements the client's therapist made to you about the client's mental
health.

Question 5
You represent Paul Payor in a tax fraud matter. You asked Paul to gather all of his accounting
records. He gathers all of his 2009 tax year records (the year in dispute) and gives these to you.
After he told the IRS that he no longer had his accounting records but had turned them all over to
you (assume that is true), the IRS issued a summons demanding that your produce "all of Paul
Payor's accounting records for the tax year 2009." (In doing so, the IRS has not made any
statements implying any wrongdoing on your behalf). You object to the summons on behalf of
your client on the grounds of attorney-client privilege. The IRS brings an action to enforce the
summons in the United States District Court, where you again renew your objection on the
grounds of attorney-client privilege. Which statement best describes the likely outcome of your
objection?

A. Your objection will be overruled and you will be required to produce the documents
because the attorney-client privilege does not apply in the enforcement of an IRS
summons.
B. Your objection will overruled and you will be required to produce the documents because
the documents were not a communication for the purposes of obtaining legal advice.
C. The objection will be sustained and you will not be required to produce the documents
because you received the documents in the course of communicating with a client for the
purposes of seeking legal advice.
D. Your objection will be sustained because the tax records are information relating to the

43
representation and you do not have the client's consent to disclose those records.

Question 6
Assume that in the Paul Payor representation, the court overrules your objections (rightly or
wrongly) and that you have no further recourse and that you must and do produce the documents
to the court. Another client would benefit significantly (without any direct harm to Paul Payor)
by having access to Paul's accounting records. May you disclose the records to the second client
without the knowledge or consent of Paul?
 
A. No, the documents are confidential and there is no exception to the duty that applies here.
B. Yes, because the court has overruled your claim of privilege, the documents are no longer
confidential.

Question 7
Your client Darren Distraught is distressed over his pending bankruptcy and has made several
comments in the past week about "putting an end to his trouble" One evening he calls you from a
local hotel and tells you in terrifying detail when, how, and why he plans to commit suicide that
evening. He is not interested in listening to your pleas for him to reconsider. You are convinced
that he will carry through on his threat if you do not get someone to stop him. Attempted suicide
is not a crime in this state; nonetheless, you decide that the best option to protect him is to call
911. The police arrive and place him in protective custody. Assuming your state has adopted
ABA Model Rule of Professional Conduct 1.6, are you subject to discipline for your disclosure?
 
A. No because his call to you was not for the purposes of seeking legal advice and so was
not confidential.
B. No because you reasonably believed that disclosure would prevent his death and calling
911 was a reasonable response.
C. Yes because the client was not threatening to commit a crime or fraud.
D. Yes because you did not first secure the client's consent to call the police.

Question 8
You are representing Jack Jamail in arranging for a sale of his business. In the course of the
representation, your independent investigation of Jack's finances causes you to realize that Jack
has lied to you about the value of his business. You had incorporated his misrepresentations into
the terms of the offer you had drafted and sent to buyers. You try to convince Jack to rescind his
fraudulent statements and he refuses. If the purchasers rely on his financials, they will suffer
substantial financial injury. you withdraw from the representation and reveal the fraud to the
buyers. Your client is unable to sell the business and goes bankrupt when the business fails.
Assume your state has adopted ABA Model Rule of Professional Conduct 1.6. Which of the
following reasons best describes why you would NOT be subject to discipline for disclosing the
client's information to the potential buyers?

A. Because the client persisted in a course of action involving your services that you
reasonably believed was criminal or fraudulent.
B. Because you reasonably believed that your disclosure was necessary to prevent
reasonably certain substantial financial injury that would result from your client's fraud,
which the client used your services to further.

44
C. Because you are required to reveal information to prevent a client's crime or fraud if your
services were used to further that crime or fraud.
D. Because you had withdrawn from the representation and so had no further obligations of
confidentiality to your former client.

Fees, Files & Property

1.  Can you trade goods or services as payment?


 Yes, if payment is reasonable (Rule 1.5)
 • So long as it’s not media rights (Rule 1.8(d))
 • And so long as you are not acquiring an interest in the subject matter of litigation (until
the lawsuit is over) (Rule 1.8(i))
 A lawyer may accept property in payment for services, such as an ownership interest in
an enterprise, providing this does not involve acquisition of a proprietary interest in the
cause of action or subject matter of the litigation contrary to Rule 1.8 (i). However, a fee
paid in property instead of money may be subject to the requirements of Rule 1.8(a)
because such fees often have the essential qualities of a business transaction with the
client. (1.5, cmt 4)

2.  Can you use billing methods other than hourly or contingent fees?
 Yes, you can combine billing methods (e.g. hourly + contingent)
 You can charge flat fees or “value billing”
 So long as the fee is not unreasonable and doesn’t interfere with the client’s right to settle
or to fire you.

3.  What is a reasonable fee?


 The test employed by the ABA codes to measure reasonableness is so vague that it is
virtually useless. The test invokes no fewer than eight multi-part factors—and the list is
expressly non-exclusive. Accordingly, on almost any given set of facts, application of the
factors to a fee could sustain a reasonable argument that the fee is justified and a
plausible argument could also be made that the fee should have been different. (see
factors in 1.5)
 1.5 is so vague it’s virtually useless, but as a practical matter it’s reasonable if:
 negotiated fairly and client understands it
 doesn’t attempt to circumvent limitations on fees
 not grossly disproportionate to the work done
 not grossly disproportionate to the value received
 not grossly disproportionate to that charged by other atty’s with similar skills
for similar work

4.  Can you discount or increase according to ability to pay?


 Yes, even though it’s not a factor in 1.5
 The rules encourage “low-bone” (Rule 6.1)
 So long as the fee is in a range of reasonableness

45
5. Why are contingent fees so controversial?
 B/c atty’s make a lot of money compared to the work they put in on one case (though
averaged across cases, their hourly rate equivalent isn’t necessarily that high);
 B/c atty’s have a stake in the outcome which creates a conflict of interest of sorts, leading
a few atty’s to stir up fraudulent litigation;
 B/c atty’s who use contingent fee agreements often sue entrenched or powerful interests
 From Book: some say they create incentives for atty’s to stir up litigation that is
unnecessary or frivolous, and that atty’s recover fees that are excessive in light of the
work they perform in the individual case.
 
6. Why can’t you charge contingent fees in a divorce case or a criminal case?
 1.5 prohibits contingent fees in criminal defense and some aspects of family law b/c of
dangers of overreaching w/ particularly vulnerable clients and b/c contingent fees might
create an incentive for attorney’s to counsel clients against plea bargains (in criminal
cases) or reconciliation (in divorce).
 Contingent fees are not necessary in either case (fee shifting in divorce and public
defender in criminal cases)
 B/c we don’t like the perverse incentives it might create (discourage reconciliation,
condone perjury)
 B/c there’s no financial “stake” in most criminal law cases and the financial “stake” in
family law cases belongs to the children

7.  What is an example of a law that puts a cap on attorneys’ fees?


 Mostly in areas of law in which we are concerned about the effects of large fees on third
parties or the public fisc (e.g. bankruptcy, workers comp)
 Some states cap fees in malpractice actions
 When govt. is paying the fee (e.g. appointed representation)
 
8. What is the lodestar method?
 Method of accounting for fees in fee shifting cases
 Reasonable hourly rate x reasonable hours is the “lodestar” – must prove any variation
 sometimes used in quantum meruit cases
 you ordinarily cannot collect from your client if you are awarded fees from the other side
 Where settlement b/w parties is not possible, the most useful starting point for court
determination of the amount of a reasonable fee [payable by the loser] is the number of
hours reasonably expended on the litigation multiplied by a reasonable hourly rate.
 
9.  Why keep time if you aren’t billing on an hourly basis?
 B/c the practice of keeping time records is an important management tool for monitoring
efficiencies, comparing productivity, and justifying fees determined on other bases.
 Allocating profits; collecting fees from others; reporting (e.g. CLE hours); defending
discipline; conflicts checking; efficiency awareness; concentration and focus
 
10. How do you keep time?

46
 Record time contemporaneously
 Use reasonable increments (beware of “rounding up”)
 Describing the activity (codes help)
 
11.What are some examples of unethical time-based billing?’
 Churning: generating unnecessary or duplicative work simply to run up hours
 + Chipping: breaking down a task into multiple small entries that add up to more than the
task should have taken
 + Double Charging: charging twice for the same task, frequently by having more than 1
person repeat the task or by charging more than one client for the same time)
 + Padding: charging for more time than is actually spent on the task, sometimes a result
of excessive “rounding up” of time segments)
 = FRAUD
 
12.What is the difference between a retainer & an advance?
 Retainer: a payment that belongs to the atty when made, goes in the atty’s account, and
represents lost opportunity or the absolute value of the atty agreeing to represent a client.
it is almost always unethical
 Advance: an advance payment of fees and goes in the client trust account until earned or
refunded. It is highly ethical if reasonable.
 
13. How do you make sure clients will pay?
 Screen clients
 Get advance payments
 Make clear agreements
 Communicate value
 Bill early and often
 Withdraw early if you anticipate problems
 Use atty’s liens in contingent fee cases
 Include ADR clauses for fee disputes
 
14. If you have a contingent fee and the client fires you before the case settles, can you still
collect?
 If client had good cause to fire you, you may have to forfeit your fees
 If no good cause, you can sue for quantum meruit (based on reasonble hours worked up
to the limit of the contingent fee), but only after the case is over (i.e., a settlement or
judgment received).
 
15.What is an IOLTA account and who has to have one?
 Is an interest bearing trust acct in which the interest is sent to the IOLTA foundation
instead of the client
 Atty’s who place their clients funds in a pooled interest-bearing account (a trust or
escrow acct) must set up an account so that the attorney pay’s the interest to a tax-
exempt, non-profit organization (IOTA’S).
o Atty’s cannot keep the interest b/c lawyers are ethically barred from benefiting

47
from the use of their clients money, including interest on it
 The pooled interest accounts are called IOLTA accounts and the IOLTA organization
administers the funds generated to provide financial assistance to various non-profit
agencies that provide legal aid to the poor
 Every attorney who has client money has an IOLTA account
 
16. Why do I have to turn over files to a client who hasn’t paid just so they can give those
files to some other attorney?
 B/c the file belongs to the client, from cover to cover, and an atty may not withhold any
portion of the file in order to enforce payment of expenses
 The balance of protecting your economic interest and protecting your client’s legal rights
has been tipped in favor of the client. So bill early and often and take care of your
clients so you aren’t in this position

17. Do I have to report another attorney’s overbilling?


 To the client? YES; b/c you have a duty under 1.4 to communicate and a fiduciary duty
 To the disciplinary authorities? Yes if it meets the requirements of 8.2 and it doesn’t
violate 1.6 (confidentiality)
 
18. How do you share fees with another attorney?
 Within a firm, however you choose (but make sure it is clear so you can avoid the kind of
fights that we saw in Welch v. Davis).
 B/w atty’s not in the same firm: Paragraph (e) of 1.5 permits the lawyers to divide a fee
either on the basis of the proportion of services they render or if each lawyer assumes
responsibility for the representation as a whole. In addition, the client must agree to the
arrangement, including the share that each lawyer is to receive, and the agreement must
be confirmed in writing. (1.5, cmt 7)
 

48

You might also like